Programs & Examples On #Openxml

Questions regarding reading and writing of Microsoft Office Open XML files either through direct manipulation of the XML files contained in a zip package or through the Microsoft Office Open XML SDK.

Export DataTable to Excel with Open Xml SDK in c#

I tried accepted answer and got message saying generated excel file is corrupted when trying to open. I was able to fix it by doing few modifications like adding below line end of the code.

workbookPart.Workbook.Save();

I have posted full code @ Export DataTable to Excel with Open XML in c#

How can I use iptables on centos 7?

I modified the /etc/sysconfig/ip6tables-config file changing:

IP6TABLES_SAVE_ON_STOP="no"

To:

IP6TABLES_SAVE_ON_STOP="yes"

And this:

IP6TABLES_SAVE_ON_RESTART="no"

To:

IP6TABLES_SAVE_ON_RESTART="yes"

This seemed to save the changes I made using the iptables commands through a reboot.

Strip / trim all strings of a dataframe

You can use the apply function of the Series object:

>>> df = pd.DataFrame([['  a  ', 10], ['  c  ', 5]])
>>> df[0][0]
'  a  '
>>> df[0] = df[0].apply(lambda x: x.strip())
>>> df[0][0]
'a'

Note the usage of strip and not the regex which is much faster

Another option - use the apply function of the DataFrame object:

>>> df = pd.DataFrame([['  a  ', 10], ['  c  ', 5]])
>>> df.apply(lambda x: x.apply(lambda y: y.strip() if type(y) == type('') else y), axis=0)

   0   1
0  a  10
1  c   5

asynchronous vs non-blocking

synchronous / asynchronous is to describe the relation between two modules.
blocking / non-blocking is to describe the situation of one module.

An example:
Module X: "I".
Module Y: "bookstore".
X asks Y: do you have a book named "c++ primer"?

  1. blocking: before Y answers X, X keeps waiting there for the answer. Now X (one module) is blocking. X and Y are two threads or two processes or one thread or one process? we DON'T know.

  2. non-blocking: before Y answers X, X just leaves there and do other things. X may come back every two minutes to check if Y has finished its job? Or X won't come back until Y calls him? We don't know. We only know that X can do other things before Y finishes its job. Here X (one module) is non-blocking. X and Y are two threads or two processes or one process? we DON'T know. BUT we are sure that X and Y couldn't be one thread.

  3. synchronous: before Y answers X, X keeps waiting there for the answer. It means that X can't continue until Y finishes its job. Now we say: X and Y (two modules) are synchronous. X and Y are two threads or two processes or one thread or one process? we DON'T know.

  4. asynchronous: before Y answers X, X leaves there and X can do other jobs. X won't come back until Y calls him. Now we say: X and Y (two modules) are asynchronous. X and Y are two threads or two processes or one process? we DON'T know. BUT we are sure that X and Y couldn't be one thread.


Please pay attention on the two bold-sentences above. Why does the bold-sentence in the 2) contain two cases whereas the bold-sentence in the 4) contains only one case? This is a key of the difference between non-blocking and asynchronous.

Here is a typical example about non-blocking & synchronous:

// thread X
while (true)
{
    msg = recv(Y, NON_BLOCKING_FLAG);
    if (msg is not empty)
    {
        break;
    }
    else
    {
        sleep(2000); // 2 sec
    }
}

// thread Y
// prepare the book for X
send(X, book);

You can see that this design is non-blocking (you can say that most of time this loop does something nonsense but in CPU's eyes, X is running, which means that X is non-blocking) whereas X and Y are synchronous because X can't continue to do any other things(X can't jump out of the loop) until it gets the book from Y.
Normally in this case, make X blocking is much better because non-blocking spends much resource for a stupid loop. But this example is good to help you understand the fact: non-blocking doesn't mean asynchronous.

The four words do make us confused easily, what we should remember is that the four words serve for the design of architecture. Learning about how to design a good architecture is the only way to distinguish them.

For example, we may design such a kind of architecture:

// Module X = Module X1 + Module X2
// Module X1
while (true)
{
    msg = recv(many_other_modules, NON_BLOCKING_FLAG);
    if (msg is not null)
    {
        if (msg == "done")
        {
            break;
        }
        // create a thread to process msg
    }
    else
    {
        sleep(2000); // 2 sec
    }
}
// Module X2
broadcast("I got the book from Y");


// Module Y
// prepare the book for X
send(X, book);

In the example here, we can say that

  • X1 is non-blocking
  • X1 and X2 are synchronous
  • X and Y are asynchronous

If you need, you can also describe those threads created in X1 with the four words.

The more important things are: when do we use synchronous instead of asynchronous? when do we use blocking instead of non-blocking? Is making X1 blocking better than non-blocking? Is making X and Y synchronous better than asynchronous? Why is Nginx non-blocking? Why is Apache blocking? These questions are what you must figure out.

To make a good choice, you must analyze your need and test the performance of different architectures. There is no such an architecture that is suitable for various of needs.

PowerShell The term is not recognized as cmdlet function script file or operable program

You first have to 'dot' source the script, so for you :

. .\Get-NetworkStatistics.ps1

The first 'dot' asks PowerShell to load the script file into your PowerShell environment, not to start it. You should also use set-ExecutionPolicy Unrestricted or set-ExecutionPolicy AllSigned see(the Execution Policy instructions).

Convert timedelta to total seconds

You can use mx.DateTime module

import mx.DateTime as mt

t1 = mt.now() 
t2 = mt.now()
print int((t2-t1).seconds)

Hiding the R code in Rmarkdown/knit and just showing the results

Sure, just do

```{r someVar, echo=FALSE}
someVariable
```

to show some (previously computed) variable someVariable. Or run code that prints etc pp.

So for plotting, I have eg

### Impact of choice of ....
```{r somePlot, echo=FALSE}
plotResults(Res, Grid, "some text", "some more text")
```

where the plotting function plotResults is from a local package.

When to use reinterpret_cast?

Here is a variant of Avi Ginsburg's program which clearly illustrates the property of reinterpret_cast mentioned by Chris Luengo, flodin, and cmdLP: that the compiler treats the pointed-to memory location as if it were an object of the new type:

#include <iostream>
#include <string>
#include <iomanip>
using namespace std;

class A
{
public:
    int i;
};

class B : public A
{
public:
    virtual void f() {}
};

int main()
{
    string s;
    B b;
    b.i = 0;
    A* as = static_cast<A*>(&b);
    A* ar = reinterpret_cast<A*>(&b);
    B* c = reinterpret_cast<B*>(ar);
    
    cout << "as->i = " << hex << setfill('0')  << as->i << "\n";
    cout << "ar->i = " << ar->i << "\n";
    cout << "b.i   = " << b.i << "\n";
    cout << "c->i  = " << c->i << "\n";
    cout << "\n";
    cout << "&(as->i) = " << &(as->i) << "\n";
    cout << "&(ar->i) = " << &(ar->i) << "\n";
    cout << "&(b.i) = " << &(b.i) << "\n";
    cout << "&(c->i) = " << &(c->i) << "\n";
    cout << "\n";
    cout << "&b = " << &b << "\n";
    cout << "as = " << as << "\n";
    cout << "ar = " << ar << "\n";
    cout << "c  = " << c  << "\n";
    
    cout << "Press ENTER to exit.\n";
    getline(cin,s);
}

Which results in output like this:

as->i = 0
ar->i = 50ee64
b.i   = 0
c->i  = 0

&(as->i) = 00EFF978
&(ar->i) = 00EFF974
&(b.i)   = 00EFF978
&(c->i)  = 00EFF978

&b = 00EFF974
as = 00EFF978
ar = 00EFF974
c  = 00EFF974
Press ENTER to exit.

It can be seen that the B object is built in memory as B-specific data first, followed by the embedded A object. The static_cast correctly returns the address of the embedded A object, and the pointer created by static_cast correctly gives the value of the data field. The pointer generated by reinterpret_cast treats b's memory location as if it were a plain A object, and so when the pointer tries to get the data field it returns some B-specific data as if it were the contents of this field.

One use of reinterpret_cast is to convert a pointer to an unsigned integer (when pointers and unsigned integers are the same size):

int i; unsigned int u = reinterpret_cast<unsigned int>(&i);

Is there a library function for Root mean square error (RMSE) in python?

from sklearn import metrics
import bumpy as np
print(no.sqrt(metrics.mean_squared_error(actual,predicted)))

Change color of Label in C#

You can try this with Color.FromArgb:

Random rnd = new Random();
lbl.ForeColor = Color.FromArgb(rnd.Next(255), rnd.Next(255), rnd.Next(255));

Get text of the selected option with jQuery

Also u can consider this

$('#select_2').find('option:selected').text();

which might be a little faster solution though I am not sure.

How do I pass a datetime value as a URI parameter in asp.net mvc?

Try to use toISOString(). It returns string in ISO8601 format.

from javascript

$.get('/example/doGet?date=' + new Date().toISOString(), function (result) {
    console.log(result);
});

from c#

[HttpGet]
public JsonResult DoGet(DateTime date)
{
    return Json(date.ToString(), JsonRequestBehavior.AllowGet);
}

Replace a value if null or undefined in JavaScript

Destructuring solution

Question content may have changed, so I'll try to answer thoroughly.

Destructuring allows you to pull values out of anything with properties. You can also define default values when null/undefined and name aliases.

const options = {
    filters : {
        firstName : "abc"
    } 
}

const {filters: {firstName = "John", lastName = "Smith"}} = options

// firstName = "abc"
// lastName = "Smith"

NOTE: Capitalization matters

If working with an array, here is how you do it.

In this case, name is extracted from each object in the array, and given its own alias. Since the object might not exist = {} was also added.

const options = {
    filters: [{
        name: "abc",
        value: "lots"
    }]
}

const {filters:[{name : filter1 = "John"} = {}, {name : filter2 = "Smith"} = {}]} = options

// filter1 = "abc"
// filter2 = "Smith"

More Detailed Tutorial

Browser Support 92% July 2020

Can I set variables to undefined or pass undefined as an argument?

Try this:

// found on UglifyJS
variable = void 0;

How can I Convert HTML to Text in C#?

If you are using .NET framework 4.5 you can use System.Net.WebUtility.HtmlDecode() which takes a HTML encoded string and returns a decoded string.

Documented on MSDN at: http://msdn.microsoft.com/en-us/library/system.net.webutility.htmldecode(v=vs.110).aspx

You can use this in a Windows Store app as well.

Easy way to convert a unicode list to a list containing python strings?

Just simply use this code

EmployeeList = eval(EmployeeList)
EmployeeList = [str(x) for x in EmployeeList]

Could not load dynamic library 'cudart64_101.dll' on tensorflow CPU-only installation

I installed cudatoolkit 11 and copy dll C:\Program Files\NVIDIA GPU Computing Toolkit\CUDA\v11.1\bin to C:\Windows\System32. It fixed for PyCharm but not for Anaconda jupyter:

[name: "/device:CPU:0" device_type: "CPU" memory_limit: 268435456 locality { } incarnation: 6812190123916921346 , name: "/device:GPU:0" device_type: "GPU" memory_limit: 13429637120 locality { bus_id: 1
links { } } incarnation: 18025633343883307728 physical_device_desc: "device: 0, name: Quadro P5000, pci bus id: 0000:02:00.0, compute capability: 6.1" ]

Throwing multiple exceptions in a method of an interface in java

You can declare as many Exceptions as you want for your interface method. But the class you gave in your question is invalid. It should read

public class MyClass implements MyInterface {
  public void find(int x) throws A_Exception, B_Exception{
    ----
    ----
    ---
  }
}

Then an interface would look like this

public interface MyInterface {
  void find(int x) throws A_Exception, B_Exception;
}

How to add title to seaborn boxplot

For a single boxplot:

import seaborn as sb
sb.boxplot(data=Array).set_title('Title')

For more boxplot in the same plot:

import seaborn as sb
sb.boxplot(data=ArrayofArray).set_title('Title')

e.g.

import seaborn as sb
myarray=[78.195229, 59.104538, 19.884109, 25.941648, 72.234825, 82.313911]
sb.boxplot(data=myarray).set_title('myTitle')

Most Useful Attributes

The DebuggerHiddenAttribute which allows to avoiding step into code which should not be debugged.

public static class CustomDebug
{
    [DebuggerHidden]
    public static void Assert(Boolean condition, Func<Exception> exceptionCreator) { ... }
}

...

// The following assert fails, and because of the attribute the exception is shown at this line
// Isn't affecting the stack trace
CustomDebug.Assert(false, () => new Exception()); 

Also it prevents from showing methods in stack trace, useful when having a method which just wraps another method:

[DebuggerHidden]
public Element GetElementAt(Vector2 position)
{
    return GetElementAt(position.X, position.Y);
}

public Element GetElementAt(Single x, Single y) { ... }

If you now call GetElementAt(new Vector2(10, 10)) and a error occurs at the wrapped method, the call stack is not showing the method which is calling the method which throws the error.

Run PHP Task Asynchronously

It's a great idea to use cURL as suggested by rojoca.

Here is an example. You can monitor text.txt while the script is running in background:

<?php

function doCurl($begin)
{
    echo "Do curl<br />\n";
    $url = 'http://'.$_SERVER['SERVER_NAME'].$_SERVER['REQUEST_URI'];
    $url = preg_replace('/\?.*/', '', $url);
    $url .= '?begin='.$begin;
    echo 'URL: '.$url.'<br>';
    $ch = curl_init();
    curl_setopt($ch, CURLOPT_URL, $url);
    curl_setopt($ch, CURLOPT_RETURNTRANSFER, true);
    $result = curl_exec($ch);
    echo 'Result: '.$result.'<br>';
    curl_close($ch);
}


if (empty($_GET['begin'])) {
    doCurl(1);
}
else {
    while (ob_get_level())
        ob_end_clean();
    header('Connection: close');
    ignore_user_abort();
    ob_start();
    echo 'Connection Closed';
    $size = ob_get_length();
    header("Content-Length: $size");
    ob_end_flush();
    flush();

    $begin = $_GET['begin'];
    $fp = fopen("text.txt", "w");
    fprintf($fp, "begin: %d\n", $begin);
    for ($i = 0; $i < 15; $i++) {
        sleep(1);
        fprintf($fp, "i: %d\n", $i);
    }
    fclose($fp);
    if ($begin < 10)
        doCurl($begin + 1);
}

?>

What is the <leader> in a .vimrc file?

The "Leader key" is a way of extending the power of VIM's shortcuts by using sequences of keys to perform a command. The default leader key is backslash. Therefore, if you have a map of <Leader>Q, you can perform that action by typing \Q.

Paste MS Excel data to SQL Server

why not just use export/import wizard in SSMS?

How to get current foreground activity context in android?

I could not find a solution that our team would be happy with so we rolled our own. We use ActivityLifecycleCallbacks to keep track of current activity and then expose it through a service. More details here: https://stackoverflow.com/a/38650587/10793

How to make a variadic macro (variable number of arguments)

C99 way, also supported by VC++ compiler.

#define FOO(fmt, ...) printf(fmt, ##__VA_ARGS__)

Multiple left joins on multiple tables in one query

You can do like this

SELECT something
FROM
    (a LEFT JOIN b ON a.a_id = b.b_id) LEFT JOIN c on a.a_aid = c.c_id
WHERE a.parent_id = 'rootID'

How do you specify the Java compiler version in a pom.xml file?

<project>
  [...]
  <build>
    [...]
    <plugins>
      <plugin>
        <groupId>org.apache.maven.plugins</groupId>
        <artifactId>maven-compiler-plugin</artifactId>
        <version>(whatever version is current)</version>
        <configuration>
          <!-- or whatever version you use -->
          <source>1.7</source>
          <target>1.7</target>
        </configuration>
      </plugin>
    </plugins>
    [...]
  </build>
  [...]
</project>

See the config page for the maven compiler plugin:

http://maven.apache.org/plugins/maven-compiler-plugin/examples/set-compiler-source-and-target.html

Oh, and: don't use Java 1.3.x, current versions are Java 1.7.x or 1.8.x

How to add colored border on cardview?

CardView extends FrameLayout, so it support foreground attribute. Using foreground attribute can also add border easily.

layout as follows:

<androidx.cardview.widget.CardView xmlns:android="http://schemas.android.com/apk/res/android"
    xmlns:app="http://schemas.android.com/apk/res-auto"
    android:id="@+id/link_card"
    android:layout_width="match_parent"
    android:layout_height="wrap_content"
    android:foreground="@drawable/bg_roundrect_ripple_light_border"
    app:cardCornerRadius="23dp"
    app:cardElevation="0dp">
</androidx.cardview.widget.CardView>

bg_roundrect_ripple_light_border.xml

<?xml version="1.0" encoding="utf-8"?>
<ripple xmlns:android="http://schemas.android.com/apk/res/android"
    android:color="@color/ripple_color_light">

    <item>
        <shape android:shape="rectangle">
            <stroke
                android:width="0.5dp"
                android:color="#DDDDDD" />
            <corners android:radius="23dp" />
        </shape>
    </item>

    <item android:id="@android:id/mask">
        <shape android:shape="rectangle">
            <corners android:radius="23dp" />
            <solid android:color="@color/background" />
        </shape>
    </item>

</ripple>

How to add data to DataGridView

first you need to add 2 columns to datagrid. you may do it at design time. see Columns property. then add rows as much as you need.

this.dataGridView1.Rows.Add("1", "XX");

How can I determine the direction of a jQuery scroll event?

Existing Solution

There could be 3 solution from this posting and other answer.

Solution 1

    var lastScrollTop = 0;
    $(window).on('scroll', function() {
        st = $(this).scrollTop();
        if(st < lastScrollTop) {
            console.log('up 1');
        }
        else {
            console.log('down 1');
        }
        lastScrollTop = st;
    });

Solution 2

    $('body').on('DOMMouseScroll', function(e){
        if(e.originalEvent.detail < 0) {
            console.log('up 2');
        }
        else {
            console.log('down 2');
        }
    });

Solution 3

    $('body').on('mousewheel', function(e){
        if(e.originalEvent.wheelDelta > 0) {
            console.log('up 3');
        }
        else {
            console.log('down 3');
        }
    });

Multi Browser Test

I couldn't tested it on Safari

chrome 42 (Win 7)

  • Solution 1
    • Up : 1 event per 1 scroll
    • Down : 1 event per 1 scroll
  • Solution 2
    • Up : Not working
    • Down : Not working
  • Solution 3
    • Up : 1 event per 1 scroll
    • Down : 1 event per 1 scroll

Firefox 37 (Win 7)

  • Solution 1
    • Up : 20 events per 1 scroll
    • Down : 20 events per 1 scroll
  • Solution 2
    • Up : Not working
    • Down : 1 event per 1 scroll
  • Solution 3
    • Up : Not working
    • Down : Not working

IE 11 (Win 8)

  • Solution 1
    • Up : 10 events per 1 scroll (side effect : down scroll occured at last)
    • Down : 10 events per 1 scroll
  • Solution 2
    • Up : Not working
    • Down : Not working
  • Solution 3
    • Up : Not working
    • Down : 1 event per 1 scroll

IE 10 (Win 7)

  • Solution 1
    • Up : 1 event per 1 scroll
    • Down : 1 event per 1 scroll
  • Solution 2
    • Up : Not working
    • Down : Not working
  • Solution 3
    • Up : 1 event per 1 scroll
    • Down : 1 event per 1 scroll

IE 9 (Win 7)

  • Solution 1
    • Up : 1 event per 1 scroll
    • Down : 1 event per 1 scroll
  • Solution 2
    • Up : Not working
    • Down : Not working
  • Solution 3
    • Up : 1 event per 1 scroll
    • Down : 1 event per 1 scroll

IE 8 (Win 7)

  • Solution 1
    • Up : 2 events per 1 scroll (side effect : down scroll occured at last)
    • Down : 2~4 events per 1 scroll
  • Solution 2
    • Up : Not working
    • Down : Not working
  • Solution 3
    • Up : 1 event per 1 scroll
    • Down : 1 event per 1 scroll

Combined Solution

I checked that side effect from IE 11 and IE 8 is come from if else statement. So, I replaced it with if else if statement as following.

From the multi browser test, I decided to use Solution 3 for common browsers and Solution 1 for firefox and IE 11.

I referred this answer to detect IE 11.

    // Detect IE version
    var iev=0;
    var ieold = (/MSIE (\d+\.\d+);/.test(navigator.userAgent));
    var trident = !!navigator.userAgent.match(/Trident\/7.0/);
    var rv=navigator.userAgent.indexOf("rv:11.0");

    if (ieold) iev=new Number(RegExp.$1);
    if (navigator.appVersion.indexOf("MSIE 10") != -1) iev=10;
    if (trident&&rv!=-1) iev=11;

    // Firefox or IE 11
    if(typeof InstallTrigger !== 'undefined' || iev == 11) {
        var lastScrollTop = 0;
        $(window).on('scroll', function() {
            st = $(this).scrollTop();
            if(st < lastScrollTop) {
                console.log('Up');
            }
            else if(st > lastScrollTop) {
                console.log('Down');
            }
            lastScrollTop = st;
        });
    }
    // Other browsers
    else {
        $('body').on('mousewheel', function(e){
            if(e.originalEvent.wheelDelta > 0) {
                console.log('Up');
            }
            else if(e.originalEvent.wheelDelta < 0) {
                console.log('Down');
            }
        });
    }

Maximum call stack size exceeded error

It means that somewhere in your code, you are calling a function which in turn calls another function and so forth, until you hit the call stack limit.

This is almost always because of a recursive function with a base case that isn't being met.

Viewing the stack

Consider this code...

(function a() {
    a();
})();

Here is the stack after a handful of calls...

Web Inspector

As you can see, the call stack grows until it hits a limit: the browser hardcoded stack size or memory exhaustion.

In order to fix it, ensure that your recursive function has a base case which is able to be met...

(function a(x) {
    // The following condition 
    // is the base case.
    if ( ! x) {
        return;
    }
    a(--x);
})(10);

How to add an image to the "drawable" folder in Android Studio?

Install and use the Android Drawable Importer plugin:

https://github.com/winterDroid/android-drawable-importer-intellij-plugin

Instructions on how to install the plugin are on that page. It's called "Android Drawable Importer" in the plugin search results.

Once installed:

  1. right click on "res" folder and select New -> Batch Drawable Import
  2. hit the + and select your source image
  3. choose what resolution you want it considered and which other sizes to auto-generate for

Seems kind of ridiculous that Android Studio doesn't support this directly.

EDIT: But Xcode doesn't either so.... :-(

Eclipse does not start when I run the exe?

eclipse.exe -clean -clearPersistedState

All you have to do is go the dir where your eclipse.exe is there and run the above command

How to compare two List<String> to each other?

You can check in all the below ways for a List

List<string> FilteredList = new List<string>();
//Comparing the two lists and gettings common elements.
FilteredList = a1.Intersect(a2, StringComparer.OrdinalIgnoreCase);

C# Call a method in a new thread

If you actually start a new thread, that thread will terminate when the method finishes:

Thread thread = new Thread(SecondFoo);
thread.Start();

Now SecondFoo will be called in the new thread, and the thread will terminate when it completes.

Did you actually mean that you wanted the thread to terminate when the method in the calling thread completes?

EDIT: Note that starting a thread is a reasonably expensive operation. Do you definitely need a brand new thread rather than using a threadpool thread? Consider using ThreadPool.QueueUserWorkItem or (preferrably, if you're using .NET 4) TaskFactory.StartNew.

What is the purpose of class methods?

I recently wanted a very light-weight logging class that would output varying amounts of output depending on the logging level that could be programmatically set. But I didn't want to instantiate the class every time I wanted to output a debugging message or error or warning. But I also wanted to encapsulate the functioning of this logging facility and make it reusable without the declaration of any globals.

So I used class variables and the @classmethod decorator to achieve this.

With my simple Logging class, I could do the following:

Logger._level = Logger.DEBUG

Then, in my code, if I wanted to spit out a bunch of debugging information, I simply had to code

Logger.debug( "this is some annoying message I only want to see while debugging" )

Errors could be out put with

Logger.error( "Wow, something really awful happened." )

In the "production" environment, I can specify

Logger._level = Logger.ERROR

and now, only the error message will be output. The debug message will not be printed.

Here's my class:

class Logger :
    ''' Handles logging of debugging and error messages. '''

    DEBUG = 5
    INFO  = 4
    WARN  = 3
    ERROR = 2
    FATAL = 1
    _level = DEBUG

    def __init__( self ) :
        Logger._level = Logger.DEBUG

    @classmethod
    def isLevel( cls, level ) :
        return cls._level >= level

    @classmethod
    def debug( cls, message ) :
        if cls.isLevel( Logger.DEBUG ) :
            print "DEBUG:  " + message

    @classmethod
    def info( cls, message ) :
        if cls.isLevel( Logger.INFO ) :
            print "INFO :  " + message

    @classmethod
    def warn( cls, message ) :
        if cls.isLevel( Logger.WARN ) :
            print "WARN :  " + message

    @classmethod
    def error( cls, message ) :
        if cls.isLevel( Logger.ERROR ) :
            print "ERROR:  " + message

    @classmethod
    def fatal( cls, message ) :
        if cls.isLevel( Logger.FATAL ) :
            print "FATAL:  " + message

And some code that tests it just a bit:

def logAll() :
    Logger.debug( "This is a Debug message." )
    Logger.info ( "This is a Info  message." )
    Logger.warn ( "This is a Warn  message." )
    Logger.error( "This is a Error message." )
    Logger.fatal( "This is a Fatal message." )

if __name__ == '__main__' :

    print "Should see all DEBUG and higher"
    Logger._level = Logger.DEBUG
    logAll()

    print "Should see all ERROR and higher"
    Logger._level = Logger.ERROR
    logAll()

What is the proper way to format a multi-line dict in Python?

Since your keys are strings and since we are talking about readability, I prefer :

mydict = dict(
    key1 = 1,
    key2 = 2,
    key3 = 3
)

How can I commit a single file using SVN over a network?

strange that your command works, i thougth it would need a target directory. but it looks like it assumes current pwd as default.

cd myapp
svn ci page1.html

you can also just do svn ci in or on that folder and it will detect all changes automatically and give you a list of what will be checked in

man svn tells you the rest

Get all column names of a DataTable into string array using (LINQ/Predicate)

I'd suggest using such extension method:

public static class DataColumnCollectionExtensions
{
    public static IEnumerable<DataColumn> AsEnumerable(this DataColumnCollection source)
    {
        return source.Cast<DataColumn>();
    }
}

And therefore:

string[] columnNames = dataTable.Columns.AsEnumerable().Select(column => column.Name).ToArray();

You may also implement one more extension method for DataTable class to reduce code:

public static class DataTableExtensions
{
    public static IEnumerable<DataColumn> GetColumns(this DataTable source)
    {
        return source.Columns.AsEnumerable();
    }
}

And use it as follows:

string[] columnNames = dataTable.GetColumns().Select(column => column.Name).ToArray();

How to create an Oracle sequence starting with max value from a table?

you might want to start with max(trans_seq_no) + 1.

watch:

SQL> create table my_numbers(my_number number not null primary key);

Table created.

SQL> insert into my_numbers(select rownum from user_objects);

260 rows created.

SQL> select max(my_number) from my_numbers;

MAX(MY_NUMBER)
--------------
           260

SQL> create sequence my_number_sn start with 260;

Sequence created.

SQL> insert into my_numbers(my_number) values (my_number_sn.NEXTVAL);
insert into my_numbers(my_number) values (my_number_sn.NEXTVAL)
*
ERROR at line 1:
ORA-00001: unique constraint (NEIL.SYS_C00102439) violated

When you create a sequence with a number, you have to remember that the first time you select against the sequence, Oracle will return the initial value that you assigned it.

SQL> drop sequence my_number_sn;

Sequence dropped.

SQL> create sequence my_number_sn start with 261;

Sequence created.

SQL>  insert into my_numbers(my_number) values (my_number_sn.NEXTVAL);

1 row created.

If you're trying to do the 'gapless' thing, I strongly advise you to

1 not do it, and #2 not use a sequence for it.

When to use an interface instead of an abstract class and vice versa?

One interesting location where interfaces fare better than abstract classes is when you need to add extra functionality to a group of (related or unrelated) objects. If you cannot give them a base abstract class (e.g., they are sealed or already have a parent), you can give them a dummy (empty) interface instead, and then simply write extension methods for that interface.

$this->session->set_flashdata() and then $this->session->flashdata() doesn't work in codeigniter

// Set flash data 
$this->session->set_flashdata('message_name', 'This is my message');
// After that you need to used redirect function instead of load view such as 
redirect("admin/signup");

// Get Flash data on view 
$this->session->flashdata('message_name');

How to convert vector to array

Vectors effectively are arrays under the skin. If you have a function:

void f( double a[]);

you can call it like this:

vector <double> v;
v.push_back( 1.23 )
f( &v[0] );

You should not ever need to convert a vector into an actual array instance.

HttpClient won't import in Android Studio

Add these two lines under dependencies

compile 'org.apache.httpcomponents:httpcore:4.4.1'
compile 'org.apache.httpcomponents:httpclient:4.5'

then

useLibrary 'org.apache.http.legacy'

under the android

How can I generate an ObjectId with mongoose?

You can find the ObjectId constructor on require('mongoose').Types. Here is an example:

var mongoose = require('mongoose');
var id = mongoose.Types.ObjectId();

id is a newly generated ObjectId.

You can read more about the Types object at Mongoose#Types documentation.

How to comment out a block of Python code in Vim

I have the following lines in my .vimrc:

" comment line, selection with Ctrl-N,Ctrl-N
au BufEnter *.py nnoremap  <C-N><C-N>    mn:s/^\(\s*\)#*\(.*\)/\1#\2/ge<CR>:noh<CR>`n
au BufEnter *.py inoremap  <C-N><C-N>    <C-O>mn<C-O>:s/^\(\s*\)#*\(.*\)/\1#\2/ge<CR><C-O>:noh<CR><C-O>`n
au BufEnter *.py vnoremap  <C-N><C-N>    mn:s/^\(\s*\)#*\(.*\)/\1#\2/ge<CR>:noh<CR>gv`n

" uncomment line, selection with Ctrl-N,N
au BufEnter *.py nnoremap  <C-N>n     mn:s/^\(\s*\)#\([^ ]\)/\1\2/ge<CR>:s/^#$//ge<CR>:noh<CR>`n
au BufEnter *.py inoremap  <C-N>n     <C-O>mn<C-O>:s/^\(\s*\)#\([^ ]\)/\1\2/ge<CR><C-O>:s/^#$//ge<CR><C-O>:noh<CR><C-O>`n
au BufEnter *.py vnoremap  <C-N>n     mn:s/^\(\s*\)#\([^ ]\)/\1\2/ge<CR>gv:s/#\n/\r/ge<CR>:noh<CR>gv`n

The shortcuts preserve your cursor position and your comments as long as they start with # (there is space after #). For example:

# variable x
x = 0

After commenting:

# variable x
#x = 0

After uncomennting:

# variable x
x = 0

Ubuntu - Run command on start-up with "sudo"

You can add the command in the /etc/rc.local script that is executed at the end of startup.

Write the command before exit 0. Anything written after exit 0 will never be executed.

@AspectJ pointcut for all methods of a class with specific annotation

From Spring's AnnotationTransactionAspect:

/**
 * Matches the execution of any public method in a type with the Transactional
 * annotation, or any subtype of a type with the Transactional annotation.
 */
private pointcut executionOfAnyPublicMethodInAtTransactionalType() :
    execution(public * ((@Transactional *)+).*(..)) && within(@Transactional *);

flutter remove back button on appbar

You can remove the back button by passing an empty new Container() as the leading argument to your AppBar.

If you find yourself doing this, you probably don't want the user to be able to press the device's back button to get back to the earlier route. Instead of calling pushNamed, try calling Navigator.pushReplacementNamed to cause the earlier route to disappear.

The function pushReplacementNamed will remove the previous route in the backstack and replace it with the new route.

Full code sample for the latter is below.

import 'package:flutter/material.dart';

class LogoutPage extends StatelessWidget {
  @override
  Widget build(BuildContext context) {
    return new Scaffold(
      appBar: new AppBar(
        title: new Text("Logout Page"),
      ),
      body: new Center(
        child: new Text('You have been logged out'),
      ),
    );
  }

}
class MyHomePage extends StatelessWidget {

  @override
  Widget build(BuildContext context) {
    return new Scaffold(
      appBar: new AppBar(
        title: new Text("Remove Back Button"),
      ),
      floatingActionButton: new FloatingActionButton(
        child: new Icon(Icons.fullscreen_exit),
        onPressed: () {
          Navigator.pushReplacementNamed(context, "/logout");
        },
      ),
    );
  }
}

void main() {
  runApp(new MyApp());
}

class MyApp extends StatelessWidget {
  @override
  Widget build(BuildContext context) {
    return new MaterialApp(
      title: 'Flutter Demo',
      home: new MyHomePage(),
      routes: {
        "/logout": (_) => new LogoutPage(),
      },
    );
  }
}

How to run test methods in specific order in JUnit4?

If you want to run test methods in a specific order in JUnit 5, you can use the below code.

@TestMethodOrder(MethodOrderer.OrderAnnotation.class)
public class MyClassTest { 

    @Test
    @Order(1)
    public void test1() {}

    @Test
    @Order(2)
    public void test2() {}

}

Uncaught ReferenceError: jQuery is not defined

you need to put it after wp_head(); Because that loads your jQuery and you need to load jQuery first and then your js

Cannot apply indexing with [] to an expression of type 'System.Collections.Generic.IEnumerable<>

The IEnumerable<T> interface does not include an indexer, you're probably confusing it with IList<T>

If the object really is an IList<T> (e.g. List<T> or an array T[]), try making the reference to it of type IList<T> too.

Otherwise, you can use myEnumerable.ElementAt(index) which uses the Enumerable.ElementAt extension method. This should work for all IEnumerable<T>s . Note that unless the (run-time) object implements IList<T>, this will cause all of the first index + 1 items to be enumerated, with all but the last being discarded.

EDIT: As an explanation, IEnumerable<T> is simply an interface that represents "that which exposes an enumerator." A concrete implementation may well be some sort of in-memory list that does allow fast-access by index, or it may not. For instance, it could be a collection that cannot efficiently satisfy such a query, such as a linked-list (as mentioned by James Curran). It may even be no sort of in-memory data-structure at all, such as an iterator, where items are generated ('yielded') on demand, or by an enumerator that fetches the items from some remote data-source. Because IEnumerable<T> must support all these cases, indexers are excluded from its definition.

How do relative file paths work in Eclipse?

You need "src/Hankees.txt"

Your file is in the source folder which is not counted as the working directory.\

Or you can move the file up to the root directory of your project and just use "Hankees.txt"

java.lang.NullPointerException: Attempt to invoke virtual method on a null object reference

Your app is crashing at:

welcomePlayer.setText("Welcome Back, " + String.valueOf(mPlayer.getName(this)) + " !");

because mPlayer=null.

You forgot to initialize Player mPlayer in your PlayGame Activity.

mPlayer = new Player(context,"");

set pythonpath before import statements

As also noted in the docs here.
Go to Python X.X/Lib and add these lines to the site.py there,

import sys
sys.path.append("yourpathstring")

This changes your sys.path so that on every load, it will have that value in it..

As stated here about site.py,

This module is automatically imported during initialization. Importing this module will append site-specific paths to the module search path and add a few builtins.

For other possible methods of adding some path to sys.path see these docs

Uses of content-disposition in an HTTP response header

Well, it seems that the Content-Disposition header was originally created for e-mail, not the web. (Link to relevant RFC.)

I'm guessing that web browsers may respond to

Response.AppendHeader("content-disposition", "inline; filename=" + fileName);

when saving, but I'm not sure.

Array of an unknown length in C#

As detailed above, the generic List<> is the best way of doing it.

If you're stuck in .NET 1.*, then you will have to use the ArrayList class instead. This does not have compile-time type checking and you also have to add casting - messy.

Successive versions have also implemented various variations - including thread safe variants.

Create list of single item repeated N times

As others have pointed out, using the * operator for a mutable object duplicates references, so if you change one you change them all. If you want to create independent instances of a mutable object, your xrange syntax is the most Pythonic way to do this. If you are bothered by having a named variable that is never used, you can use the anonymous underscore variable.

[e for _ in xrange(n)]

Test whether string is a valid integer

For me, the simplest solution was to use the variable inside a (()) expression, as so:

if ((VAR > 0))
then
  echo "$VAR is a positive integer."
fi

Of course, this solution is only valid if a value of zero doesn't make sense for your application. That happened to be true in my case, and this is much simpler than the other solutions.

As pointed out in the comments, this can make you subject to a code execution attack: The (( )) operator evaluates VAR, as stated in the Arithmetic Evaluation section of the bash(1) man page. Therefore, you should not use this technique when the source of the contents of VAR is uncertain (nor should you use ANY other form of variable expansion, of course).

Converting HTML string into DOM elements?

Check out John Resig's pure JavaScript HTML parser.

EDIT: if you want the browser to parse the HTML for you, innerHTML is exactly what you want. From this SO question:

var tempDiv = document.createElement('div');
tempDiv.innerHTML = htmlString;

How to force a WPF binding to refresh?

You can use binding expressions:

private void ComboBox_Loaded(object sender, RoutedEventArgs e)
{
    ((ComboBox)sender).GetBindingExpression(ComboBox.ItemsSourceProperty)
                      .UpdateTarget();
}

But as Blindmeis noted you can also fire change notifications, further if your collection implements INotifyCollectionChanged (for example implemented in the ObservableCollection<T>) it will synchronize so you do not need to do any of this.

Get the Selected value from the Drop down box in PHP

Couldn't you just pass the a name attribute and wrap it in a form?

<form id="form" action="do_stuff.php" method="post">
    <select id="select_catalog" name="select_catalog_query">
    <?php <<<INSERT THE SELECT OPTION LOOP>>> ?>
    </select>
</form>

And then look for $_POST['select_catalog_query'] ?

Missing styles. Is the correct theme chosen for this layout?

Just need click button 'AppTheme', and choose 'Manifest Themes'. It works in most cases.

Is there an easy way to reload css without reloading the page?

One more jQuery solution

For a single stylesheet with id "css" try this:

$('#css').replaceWith('<link id="css" rel="stylesheet" href="css/main.css?t=' + Date.now() + '"></link>');

Wrap it in a function that has global scrope and you can use it from the Developer Console in Chrome or Firebug in Firefox:

var reloadCSS = function() {
  $('#css').replaceWith('<link id="css" rel="stylesheet" href="css/main.css?t=' + Date.now() + '"></link>');
};

Remove or uninstall library previously added : cocoapods

Remove pod name from Podfile then Open Terminal, set project folder path and Run pod update command.

NOTE: pod update will update all the libraries to the latest version and will also remove those libraries whose name have been removed from podfile.

How to get the error message from the error code returned by GetLastError()?

In general, you need to use FormatMessage to convert from a Win32 error code to text.

From the MSDN documentation:

Formats a message string. The function requires a message definition as input. The message definition can come from a buffer passed into the function. It can come from a message table resource in an already-loaded module. Or the caller can ask the function to search the system's message table resource(s) for the message definition. The function finds the message definition in a message table resource based on a message identifier and a language identifier. The function copies the formatted message text to an output buffer, processing any embedded insert sequences if requested.

The declaration of FormatMessage:

DWORD WINAPI FormatMessage(
  __in      DWORD dwFlags,
  __in_opt  LPCVOID lpSource,
  __in      DWORD dwMessageId, // your error code
  __in      DWORD dwLanguageId,
  __out     LPTSTR lpBuffer,
  __in      DWORD nSize,
  __in_opt  va_list *Arguments
);

How may I reference the script tag that loaded the currently-executing script?

Here's a bit of a polyfill that leverages document.CurrentScript if it exists and falls back to finding the script by ID.

<script id="uniqueScriptId">
    (function () {
        var thisScript = document.CurrentScript || document.getElementByID('uniqueScriptId');

        // your code referencing thisScript here
    ());
</script>

If you include this at the top of every script tag I believe you'll be able to consistently know which script tag is being fired, and you'll also be able to reference the script tag in the context of an asynchronous callback.

Untested, so leave feedback for others if you try it.

How do I split a string so I can access item x?

You may find the solution in SQL User Defined Function to Parse a Delimited String helpful (from The Code Project).

You can use this simple logic:

Declare @products varchar(200) = '1|20|3|343|44|6|8765'
Declare @individual varchar(20) = null

WHILE LEN(@products) > 0
BEGIN
    IF PATINDEX('%|%', @products) > 0
    BEGIN
        SET @individual = SUBSTRING(@products,
                                    0,
                                    PATINDEX('%|%', @products))
        SELECT @individual

        SET @products = SUBSTRING(@products,
                                  LEN(@individual + '|') + 1,
                                  LEN(@products))
    END
    ELSE
    BEGIN
        SET @individual = @products
        SET @products = NULL
        SELECT @individual
    END
END

pgadmin4 : postgresql application server could not be contacted.

Just click on that pgadmin 4 icon and run as administrator. Allow the access permissions. It will start locally.

CSS table-cell equal width

Replace

  <div style="display:table;">
    <div style="display:table-cell;"></div>
    <div style="display:table-cell;"></div>
  </div>

with

  <table>
    <tr><td>content cell1</td></tr>
    <tr><td>content cell1</td></tr>
  </table>

Look at all the issues surrounding trying to make divs perform like tables. They had to add table-xxx to mimic table layouts

Tables are supported and work very well in all browsers. Why ditch them? the fact that they had to mimic them is proof they did their job and well.

In my opinion use the best tool for the job and if you want tabulated data or something that resembles tabulated data tables just work.

Very Late reply I know but worth voicing.

Command-line Unix ASCII-based charting / plotting tool

feedgnuplot is another front end to gnuplot, which handles piping in data.

    $ seq 5 | awk '{print 2*$1, $1*$1}' |
      feedgnuplot --lines --points --legend 0 "data 0" --title "Test plot" --y2 1
                  --terminal 'dumb 80,40' --exit

                                     Test plot

     10 ++------+--------+-------+-------+-------+--------+-------+------*A 25
        +       +        +       +       +       +        +       +    **#+
        |       :        :       :       :       :        : data 0+**A*** |
        |       :        :       :       :       :        :       :** #   |
      9 ++.......................................................**.##....|
        |       :        :       :       :       :        :    ** :#      |
        |       :        :       :       :       :        :  **   #       |
        |       :        :       :       :       :        :**   ##:      ++ 20
      8 ++................................................A....#..........|
        |       :        :       :       :       :      **:   #   :       |
        |       :        :       :       :       :    **  : ##    :       |
        |       :        :       :       :       :  **    :#      :       |
        |       :        :       :       :       :**      B       :       |
      7 ++......................................**......##................|
        |       :        :       :       :    ** :    ##  :       :      ++ 15
        |       :        :       :       :  **   :   #    :       :       |
        |       :        :       :       :**     : ##     :       :       |
      6 ++..............................*A.......##.......................|
        |       :        :       :    ** :     ##:        :       :       |
        |       :        :       :  **   :    #  :        :       :       |
        |       :        :       :**     :  ##   :        :       :      ++ 10
      5 ++......................**........##..............................|
        |       :        :    ** :      #B       :        :       :       |
        |       :        :  **   :    ## :       :        :       :       |
        |       :        :**     :  ##   :       :        :       :       |
      4 ++...............A.......###......................................|
        |       :      **:     ##:       :       :        :       :       |
        |       :    **  :   ##  :       :       :        :       :      ++ 5
        |       :  **    : ##    :       :       :        :       :       |
        |       :**    ##B#      :       :       :        :       :       |
      3 ++.....**..####...................................................|
        |    **####      :       :       :       :        :       :       |
        |  **## :        :       :       :       :        :       :       |
        B**     +        +       +       +       +        +       +       +
      2 A+------+--------+-------+-------+-------+--------+-------+------++ 0
        1      1.5       2      2.5      3      3.5       4      4.5      5

Printing Python version in output

Try

python --version 

or

python -V

This will return a current python version in terminal.

Sorting HTML table with JavaScript

I wrote up some code that will sort a table by a row, assuming only one <tbody> and cells don't have a colspan.

function sortTable(table, col, reverse) {
    var tb = table.tBodies[0], // use `<tbody>` to ignore `<thead>` and `<tfoot>` rows
        tr = Array.prototype.slice.call(tb.rows, 0), // put rows into array
        i;
    reverse = -((+reverse) || -1);
    tr = tr.sort(function (a, b) { // sort rows
        return reverse // `-1 *` if want opposite order
            * (a.cells[col].textContent.trim() // using `.textContent.trim()` for test
                .localeCompare(b.cells[col].textContent.trim())
               );
    });
    for(i = 0; i < tr.length; ++i) tb.appendChild(tr[i]); // append each row in order
}
// sortTable(tableNode, columId, false);

If you don't want to make the assumptions above, you'd need to consider how you want to behave in each circumstance. (e.g. put everything into one <tbody> or add up all the preceeding colspan values, etc.)

You could then attach this to each of your tables, e.g. assuming titles are in <thead>

function makeSortable(table) {
    var th = table.tHead, i;
    th && (th = th.rows[0]) && (th = th.cells);
    if (th) i = th.length;
    else return; // if no `<thead>` then do nothing
    while (--i >= 0) (function (i) {
        var dir = 1;
        th[i].addEventListener('click', function () {sortTable(table, i, (dir = 1 - dir))});
    }(i));
}

function makeAllSortable(parent) {
    parent = parent || document.body;
    var t = parent.getElementsByTagName('table'), i = t.length;
    while (--i >= 0) makeSortable(t[i]);
}

and then invoking makeAllSortable onload.


Example fiddle of it working on a table.

Finding CN of users in Active Directory

CN refers to class name, so put in your LDAP query CN=Users. Should work.

Invalid configuration object. Webpack has been initialised using a configuration object that does not match the API schema

For the people like myself, who started recently: The loaders keyword is replaced with rules; even though it still represents the concept of loaders. So my webpack.config.js, for a React app, is as follows:

var webpack = require('webpack');
var path = require('path');

var BUILD_DIR = path.resolve(__dirname, 'src/client/public');
var APP_DIR = path.resolve(__dirname, 'src/client/app');

var config = {
  entry: APP_DIR + '/index.jsx',
  output: {
    path: BUILD_DIR,
    filename: 'bundle.js'
  },
  module : {
    rules : [
      {
        test : /\.jsx?/,
        include : APP_DIR,
        loader : 'babel-loader'
      }
    ]
  }
};

module.exports = config;

How to Alter a table for Identity Specification is identity SQL Server

You cannot "convert" an existing column into an IDENTITY column - you will have to create a new column as INT IDENTITY:

ALTER TABLE ProductInProduct 
ADD NewId INT IDENTITY (1, 1);

Update:

OK, so there is a way of converting an existing column to IDENTITY. If you absolutely need this - check out this response by Martin Smith with all the gory details.

error: use of deleted function

The error message clearly says that the default constructor has been deleted implicitly. It even says why: the class contains a non-static, const variable, which would not be initialized by the default ctor.

class X {
    const int x;
};

Since X::x is const, it must be initialized -- but a default ctor wouldn't normally initialize it (because it's a POD type). Therefore, to get a default ctor, you need to define one yourself (and it must initialize x). You can get the same kind of situation with a member that's a reference:

class X { 
    whatever &x;
};

It's probably worth noting that both of these will also disable implicit creation of an assignment operator as well, for essentially the same reason. The implicit assignment operator normally does members-wise assignment, but with a const member or reference member, it can't do that because the member can't be assigned. To make assignment work, you need to write your own assignment operator.

This is why a const member should typically be static -- when you do an assignment, you can't assign the const member anyway. In a typical case all your instances are going to have the same value so they might as well share access to a single variable instead of having lots of copies of a variable that will all have the same value.

It is possible, of course, to create instances with different values though -- you (for example) pass a value when you create the object, so two different objects can have two different values. If, however, you try to do something like swapping them, the const member will retain its original value instead of being swapped.

Parsing JSON Object in Java

1.) Create an arraylist of appropriate type, in this case i.e String

2.) Create a JSONObject while passing your string to JSONObject constructor as input

  • As JSONObject notation is represented by braces i.e {}
  • Where as JSONArray notation is represented by square brackets i.e []

3.) Retrieve JSONArray from JSONObject (created at 2nd step) using "interests" as index.

4.) Traverse JASONArray using loops upto the length of array provided by length() function

5.) Retrieve your JSONObjects from JSONArray using getJSONObject(index) function

6.) Fetch the data from JSONObject using index '"interestKey"'.

Note : JSON parsing uses the escape sequence for special nested characters if the json response (usually from other JSON response APIs) contains quotes (") like this

`"{"key":"value"}"`

should be like this

`"{\"key\":\"value\"}"`

so you can use JSONParser to achieve escaped sequence format for safety as

JSONParser parser = new JSONParser();
JSONObject json = (JSONObject) parser.parse(inputString);

Code :

JSONParser parser = new JSONParser();
String response = "{interests : [{interestKey:Dogs}, {interestKey:Cats}]}";

JSONObject jsonObj = (JSONObject) parser.parse(response);

or

JSONObject jsonObj = new JSONObject("{interests : [{interestKey:Dogs}, {interestKey:Cats}]}");

List<String> interestList = new ArrayList<String>();

JSONArray jsonArray = jsonObj.getJSONArray("interests");

for(int i = 0 ; i < jsonArray.length() ; i++){
    interestList.add(jsonArray.getJSONObject(i).optString("interestKey"));
}

Note : Sometime you may see some exceptions when the values are not available in appropriate type or is there is no mapping key so in those cases when you are not sure about the presence of value so use optString, optInt, optBoolean etc which will simply return the default value if it is not present and even try to convert value to int if it is of string type and vice-versa so Simply No null or NumberFormat exceptions at all in case of missing key or value

From docs

Get an optional string associated with a key. It returns the defaultValue if there is no such key.

 public String optString(String key, String defaultValue) {
  String missingKeyValue = json_data.optString("status","N/A");
  // note there is no such key as "status" in response
  // will return "N/A" if no key found 

or To get empty string i.e "" if no key found then simply use

  String missingKeyValue = json_data.optString("status");
  // will return "" if no key found where "" is an empty string

Further reference to study

Calling a function every 60 seconds

Call a Javascript function every 2 second continuously for 10 second.

_x000D_
_x000D_
var intervalPromise;_x000D_
$scope.startTimer = function(fn, delay, timeoutTime) {_x000D_
    intervalPromise = $interval(function() {_x000D_
        fn();_x000D_
        var currentTime = new Date().getTime() - $scope.startTime;_x000D_
        if (currentTime > timeoutTime){_x000D_
            $interval.cancel(intervalPromise);_x000D_
          }                  _x000D_
    }, delay);_x000D_
};_x000D_
_x000D_
$scope.startTimer(hello, 2000, 10000);_x000D_
_x000D_
hello(){_x000D_
  console.log("hello");_x000D_
}
_x000D_
_x000D_
_x000D_

Stack Memory vs Heap Memory

Stack memory is specifically the range of memory that is accessible via the Stack register of the CPU. The Stack was used as a way to implement the "Jump-Subroutine"-"Return" code pattern in assembly language, and also as a means to implement hardware-level interrupt handling. For instance, during an interrupt, the Stack was used to store various CPU registers, including Status (which indicates the results of an operation) and Program Counter (where was the CPU in the program when the interrupt occurred).

Stack memory is very much the consequence of usual CPU design. The speed of its allocation/deallocation is fast because it is strictly a last-in/first-out design. It is a simple matter of a move operation and a decrement/increment operation on the Stack register.

Heap memory was simply the memory that was left over after the program was loaded and the Stack memory was allocated. It may (or may not) include global variable space (it's a matter of convention).

Modern pre-emptive multitasking OS's with virtual memory and memory-mapped devices make the actual situation more complicated, but that's Stack vs Heap in a nutshell.

Node.js - EJS - including a partial

In Express 4.x I used the following to load ejs:

  var path = require('path');

  // Set the default templating engine to ejs
  app.set('view engine', 'ejs');
  app.set('views', path.join(__dirname, 'views'));

  // The views/index.ejs exists in the app directory
  app.get('/hello', function (req, res) {
    res.render('index', {title: 'title'});
  });

Then you just need two files to make it work - views/index.ejs:

<%- include partials/navigation.ejs %>

And the views/partials/navigation.ejs:

<ul><li class="active">...</li>...</ul>

You can also tell Express to use ejs for html templates:

var path = require('path');
var EJS  = require('ejs');

app.engine('html', EJS.renderFile);

// Set the default templating engine to ejs
app.set('view engine', 'ejs');
app.set('views', path.join(__dirname, 'views'));

// The views/index.html exists in the app directory
app.get('/hello', function (req, res) {
  res.render('index.html', {title: 'title'});
});

Finally you can also use the ejs layout module:

var EJSLayout = require('express-ejs-layouts');
app.use(EJSLayout);

This will use the views/layout.ejs as your layout.

Write to rails console

As other have said, you want to use either puts or p. Why? Is that magic?

Actually not. A rails console is, under the hood, an IRB, so all you can do in IRB you will be able to do in a rails console. Since for printing in an IRB we use puts, we use the same command for printing in a rails console.

You can actually take a look at the console code in the rails source code. See the require of irb? :)

How to custom switch button?

I use this approach to create a custom switch using a RadioGroup and RadioButton;

Preview

enter image description here

Color Resource

<color name="blue">#FF005a9c</color>
<color name="lightBlue">#ff6691c4</color>
<color name="lighterBlue">#ffcdd8ec</color>
<color name="controlBackground">#ffffffff</color>

control_switch_color_selector (in res/color folder)

<selector xmlns:android="http://schemas.android.com/apk/res/android">
    <item
        android:state_checked="true"
        android:color="@color/controlBackground"
    />
    <item
        android:state_pressed="true"
        android:color="@color/controlBackground"
        />
    <item
        android:color="@color/blue"
        />
</selector>

Drawables

control_switch_background_border.xml

<shape xmlns:android="http://schemas.android.com/apk/res/android"
    android:shape="rectangle">
    <corners android:radius="5dp" />
    <solid android:color="@android:color/transparent" />
    <stroke
        android:width="3dp"
        android:color="@color/blue" />
</shape>

control_switch_background_selector.xml

<selector xmlns:android="http://schemas.android.com/apk/res/android">
    <item android:state_checked="true">
        <shape>
            <solid android:color="@color/blue"></solid>
        </shape>
    </item>
    <item android:state_pressed="true">
        <shape>
            <solid android:color="@color/lighterBlue"></solid>
        </shape>
    </item>
    <item>
        <shape>
            <solid android:color="@android:color/transparent"></solid>
        </shape>
    </item>
</selector>

control_switch_background_selector_middle.xml

<selector xmlns:android="http://schemas.android.com/apk/res/android">
    <item android:state_checked="true">
        <shape>
            <solid android:color="@color/blue"></solid>
        </shape>
    </item>
    <item android:state_pressed="true">
        <shape>
            <solid android:color="@color/lighterBlue"></solid>
        </shape>
    </item>
    <item>
        <layer-list>
            <item android:top="-1dp" android:bottom="-1dp" android:left="-1dp">
                <shape>
                    <solid android:color="@android:color/transparent"></solid>
                    <stroke android:width="1dp" android:color="@color/blue"></stroke>
                </shape>
            </item>
        </layer-list>
    </item>
</selector>

Layout

<RadioGroup
        android:checkedButton="@+id/calm"
        android:id="@+id/toggle"
        android:layout_width="match_parent"
        android:layout_height="wrap_content"
        android:layout_marginLeft="24dp"
        android:layout_marginRight="24dp"
        android:layout_marginBottom="24dp"
        android:layout_marginTop="24dp"
        android:background="@drawable/control_switch_background_border"
        android:orientation="horizontal">
        <RadioButton
            android:layout_marginTop="3dp"
            android:layout_marginBottom="3dp"
            android:layout_marginLeft="3dp"
            android:paddingTop="16dp"
            android:paddingBottom="16dp"
            android:id="@+id/calm"
            android:background="@drawable/control_switch_background_selector_middle"
            android:layout_width="0dp"
            android:layout_height="match_parent"
            android:layout_weight="1"
            android:button="@null"
            android:gravity="center"
            android:text="Calm"
            android:fontFamily="sans-serif-medium"
            android:textColor="@color/control_switch_color_selector"/>
        <RadioButton
            android:layout_marginTop="3dp"
            android:layout_marginBottom="3dp"
            android:paddingTop="16dp"
            android:paddingBottom="16dp"
            android:id="@+id/rumor"
            android:background="@drawable/control_switch_background_selector_middle"
            android:layout_width="0dp"
            android:layout_height="match_parent"
            android:layout_weight="1"
            android:button="@null"
            android:gravity="center"
            android:text="Rumor"
            android:fontFamily="sans-serif-medium"
            android:textColor="@color/control_switch_color_selector"/>
        <RadioButton
            android:layout_marginTop="3dp"
            android:layout_marginBottom="3dp"
            android:layout_marginRight="3dp"
            android:paddingTop="16dp"
            android:paddingBottom="16dp"
            android:id="@+id/outbreak"
            android:layout_width="0dp"
            android:layout_height="match_parent"
            android:layout_weight="1"
            android:background="@drawable/control_switch_background_selector"
            android:button="@null"
            android:gravity="center"
            android:text="Outbreak"
            android:fontFamily="sans-serif-medium"
            android:textColor="@color/control_switch_color_selector" />
</RadioGroup>

Creating hard and soft links using PowerShell

Try junction.exe

The Junction command line utility from SysInternals makes creating and deleting junctions easy.

Further reading

  • MS Terminology: soft != symbolic
    Microsoft uses "soft link" as another name for "junction".
    However: a "symbolic link" is something else entirely.
    See MSDN: Hard Links and Junctions in Windows.
    (This is in direct contradiction to the general usage of those terms where "soft link" and "symbolic link" ("symlink") DO mean the same thing.)

What is so bad about singletons?

My answer on how Singletons are bad is always, "they are hard to do right". Many of the foundational components of languages are singletons (classes, functions, namespaces and even operators), as are components in other aspects of computing (localhost, default route, virtual filesystem, etc.), and it is not by accident. While they cause trouble and frustration from time to time, they also can make a lot of things work a LOT better.

The two biggest screw ups I see are: treating it like a global & failing to define the Singleton closure.

Everyone talks about Singleton's as globals, because they basically are. However, much (sadly, not all) of the badness in a global comes not intrinsically from being global, but how you use it. Same goes for Singletons. Actually more so as "single instance" really doesn't need to mean "globally accessible". It is more a natural byproduct, and given all the bad that we know comes from it, we shouldn't be in such a hurry to exploit global accessibility. Once programmers see a Singleton they seem to always access it directly through its instance method. Instead, you should navigate to it just like you would any other object. Most code shouldn't even be aware it is dealing with a Singleton (loose coupling, right?). If only a small bit of code accesses the object like it is a global, a lot of harm is undone. I recommend enforcing it by restricting access to the instance function.

The Singleton context is also really important. The defining characteristic of a Singleton is that there is "only one", but the truth is it is "only one" within some kind of context/namespace. They are usually one of: one per thread, process, IP address or cluster, but can also be one per processor, machine, language namespace/class loader/whatever, subnet, Internet, etc.

The other, less common, mistake is to ignore the Singleton lifestyle. Just because there is only one doesn't mean a Singleton is some omnipotent "always was and always will be", nor is it generally desirable (objects without a begin and end violate all kinds of useful assumptions in code, and should be employed only in the most desperate of circumstances.

If you avoid those mistakes, Singletons can still be a PITA, bit it is ready to see a lot of the worst problems are significantly mitigated. Imagine a Java Singleton, that is explicitly defined as once per classloader (which means it needs a thread safety policy), with defined creation and destruction methods and a life cycle that dictates when and how they get invoked, and whose "instance" method has package protection so it is generally accessed through other, non-global objects. Still a potential source of trouble, but certainly much less trouble.

Sadly, rather than teaching good examples of how to do Singletons. We teach bad examples, let programmers run off using them for a while, and then tell them they are a bad design pattern.

How to calculate the IP range when the IP address and the netmask is given?

Invert mask (XOR with ones), AND it with IP. Add 1. This will be the starting range. OR IP with mask. This will be the ending range.

Take a list of numbers and return the average

You can use python's built-in function sum

  • sum will return the sum of all the values
  • len to get list's length

code:

>>> list = [1,2,3,4]
>>> sum(list)
>>> 10
>>> len(list)
>>> 4
>>> avg = float(sum(list))/len(list)
>>> 2.5
>>>"""In pyton3 don't want to specify float"""
>>> 10 / 4
>>> 2.5 

Use float because when using python 2.x, because:

  • int/int returns int value (i.e. 2)
  • float/int returns float value (i.e. 2.5)

While in Python 3.x:

  • int/int return float
  • int//int return int

.NET Core vs Mono

You have chosen not only a realistic path, but arguably one of the best ecosystems strongly backed(also X-platforms) by MS. Still you should consider following points:

  • Update: Main doc about .Net platform standard is here: https://github.com/dotnet/corefx/blob/master/Documentation/architecture/net-platform-standard.md
  • Update: Current Mono 4.4.1 cannot run latest Asp.Net core 1.0 RTM
  • Although mono is more feature complete, its future is unclear, because MS owns it for some months now and its a duplicate work for them to support it. But MS is definitely committed to .Net Core and betting big on it.
  • Although .Net core is released, the 3rd party ecosystem is not quite there. For example Nhibernate, Umbraco etc cannot run over .Net core yet. But they have a plan.
  • There are some features missing in .Net Core like System.Drawing, you should look for 3rd party libraries
  • You should use nginx as front server with kestrelserver for asp.net apps, because kestrelserver is not quite ready for production. For example HTTP/2 is not implemented.

I hope it helps

entity framework Unable to load the specified metadata resource

Craig Stuntz has written an extensive (in my opinion) blog post on troubleshooting this exact error message, I personally would start there.

The following res: (resource) references need to point to your model.

<add name="Entities" connectionString="metadata=
    res://*/Models.WraithNath.co.uk.csdl|
    res://*/Models.WraithNath.co.uk.ssdl|
    res://*/Models.WraithNath.co.uk.msl;

Make sure each one has the name of your .edmx file after the "*/", with the "edmx" changed to the extension for that res (.csdl, .ssdl, or .msl).

It also may help to specify the assembly rather than using "//*/".

Worst case, you can check everything (a bit slower but should always find the resource) by using

<add name="Entities" connectionString="metadata=
        res://*/;provider= <!-- ... -->

Compare two date formats in javascript/jquery

It's quite simple:

if(new Date(fit_start_time) <= new Date(fit_end_time))
{//compare end <=, not >=
    //your code here
}

Comparing 2 Date instances will work just fine. It'll just call valueOf implicitly, coercing the Date instances to integers, which can be compared using all comparison operators. Well, to be 100% accurate: the Date instances will be coerced to the Number type, since JS doesn't know of integers or floats, they're all signed 64bit IEEE 754 double precision floating point numbers.

How to find difference between two columns data?

select previous, Present, previous-Present as Difference from tablename

or

select previous, Present, previous-Present as Difference from #TEMP1

How to run an EXE file in PowerShell with parameters with spaces and quotes

I tried all of the suggestions but was still unable to run msiexec.exe with parameters that contained spaces. So my solution ended up using System.Diagnostics.ProcessStartInfo:

# can have spaces here, no problems
$settings = @{
  CONNECTION_STRING = "... ..."
  ENTITY_CONTEXT = "... ..."
  URL = "..."
}

$settingsJoined = ($settings.Keys | % { "$_=""$($settings[$_])""" }) -join " "
$pinfo = New-Object System.Diagnostics.ProcessStartInfo
$pinfo.WorkingDirectory = $ScriptDirectory
$pinfo.FileName = "msiexec.exe"
$pinfo.RedirectStandardError = $true
$pinfo.RedirectStandardOutput = $true
$pinfo.UseShellExecute = $false
$pinfo.Arguments = "/l* install.log /i installer.msi $settingsJoined"
$p = New-Object System.Diagnostics.Process
$p.StartInfo = $pinfo
$p.Start() | Out-Null
$p.WaitForExit()
$stdout = $p.StandardOutput.ReadToEnd()

Synchronization vs Lock

There are 4 main factors into why you would want to use synchronized or java.util.concurrent.Lock.

Note: Synchronized locking is what I mean when I say intrinsic locking.

  1. When Java 5 came out with ReentrantLocks, they proved to have quite a noticeble throughput difference then intrinsic locking. If youre looking for faster locking mechanism and are running 1.5 consider j.u.c.ReentrantLock. Java 6's intrinsic locking is now comparable.

  2. j.u.c.Lock has different mechanisms for locking. Lock interruptable - attempt to lock until the locking thread is interrupted; timed lock - attempt to lock for a certain amount of time and give up if you do not succeed; tryLock - attempt to lock, if some other thread is holding the lock give up. This all is included aside from the simple lock. Intrinsic locking only offers simple locking

  3. Style. If both 1 and 2 do not fall into categories of what you are concerned with most people, including myself, would find the intrinsic locking semenatics easier to read and less verbose then j.u.c.Lock locking.
  4. Multiple Conditions. An object you lock on can only be notified and waited for a single case. Lock's newCondition method allows for a single Lock to have mutliple reasons to await or signal. I have yet to actually need this functionality in practice, but is a nice feature for those who need it.

How can I check if a single character appears in a string?

You can use string.indexOf('a').

If the char a is present in string :

it returns the the index of the first occurrence of the character in the character sequence represented by this object, or -1 if the character does not occur.

REST HTTP status codes for failed validation or invalid duplicate

A duplicate in the database should be a 409 CONFLICT.

I recommend using 422 UNPROCESSABLE ENTITY for validation errors.

I give a longer explanation of 4xx codes here.

How to update TypeScript to latest version with npm?

Use the command where in prompt to find the current executable in path

C:\> where tsc
C:\Users\user\AppData\Roaming\npm\tsc
C:\Users\user\AppData\Roaming\npm\tsc.cmd

Determining image file size + dimensions via Javascript?

How about this:

var imageUrl = 'https://cdn.sstatic.net/Sites/stackoverflow/img/sprites.svg';
var blob = null;
var xhr = new XMLHttpRequest(); 
xhr.open('GET', imageUrl, true); 
xhr.responseType = 'blob';
xhr.onload = function() 
{
    blob = xhr.response;
    console.log(blob, blob.size);
}
xhr.send();

http://qnimate.com/javascript-create-file-object-from-url/

due to Same Origin Policy, only work under same origin

Android Image View Pinch Zooming

I made my own custom imageview with pinch to zoom. There is no limits/borders on Chirag Ravals code, so user can drag the image off the screen. This will fix it.

Here is the CustomImageView class:

    public class CustomImageVIew extends ImageView implements OnTouchListener {


    private Matrix matrix = new Matrix();
    private Matrix savedMatrix = new Matrix();

    static final int NONE = 0;
    static final int DRAG = 1;
    static final int ZOOM = 2;

    private int mode = NONE;

    private PointF mStartPoint = new PointF();
    private PointF mMiddlePoint = new PointF();
    private Point mBitmapMiddlePoint = new Point();

    private float oldDist = 1f;
    private float matrixValues[] = {0f, 0f, 0f, 0f, 0f, 0f, 0f, 0f, 0f};
    private float scale;
    private float oldEventX = 0;
    private float oldEventY = 0;
    private float oldStartPointX = 0;
    private float oldStartPointY = 0;
    private int mViewWidth = -1;
    private int mViewHeight = -1;
    private int mBitmapWidth = -1;
    private int mBitmapHeight = -1;
    private boolean mDraggable = false;


    public CustomImageVIew(Context context) {
        this(context, null, 0);
    }

    public CustomImageVIew(Context context, AttributeSet attrs) {
        this(context, attrs, 0);
    }

    public CustomImageVIew(Context context, AttributeSet attrs, int defStyle) {
        super(context, attrs, defStyle);
        this.setOnTouchListener(this);
    }

    @Override
    public void onSizeChanged (int w, int h, int oldw, int oldh){
        super.onSizeChanged(w, h, oldw, oldh);
        mViewWidth = w;
        mViewHeight = h;
    }

    public void setBitmap(Bitmap bitmap){
        if(bitmap != null){
            setImageBitmap(bitmap);

            mBitmapWidth = bitmap.getWidth();
            mBitmapHeight = bitmap.getHeight();
            mBitmapMiddlePoint.x = (mViewWidth / 2) - (mBitmapWidth /  2);
            mBitmapMiddlePoint.y = (mViewHeight / 2) - (mBitmapHeight / 2);

            matrix.postTranslate(mBitmapMiddlePoint.x, mBitmapMiddlePoint.y);
            this.setImageMatrix(matrix);
        }
    }

    @Override
    public boolean onTouch(View v, MotionEvent event){
        switch (event.getAction() & MotionEvent.ACTION_MASK) {
        case MotionEvent.ACTION_DOWN:
            savedMatrix.set(matrix);
            mStartPoint.set(event.getX(), event.getY());
            mode = DRAG;
            break;
        case MotionEvent.ACTION_POINTER_DOWN:
            oldDist = spacing(event);
            if(oldDist > 10f){
                savedMatrix.set(matrix);
                midPoint(mMiddlePoint, event);
                mode = ZOOM;
            }
            break;
        case MotionEvent.ACTION_UP:
        case MotionEvent.ACTION_POINTER_UP:
            mode = NONE;
            break;
        case MotionEvent.ACTION_MOVE:
            if(mode == DRAG){
                drag(event);
            } else if(mode == ZOOM){
                zoom(event);
            } 
            break;
        }

        return true;
    }



   public void drag(MotionEvent event){
       matrix.getValues(matrixValues);

       float left = matrixValues[2];
       float top = matrixValues[5];
       float bottom = (top + (matrixValues[0] * mBitmapHeight)) - mViewHeight;
       float right = (left + (matrixValues[0] * mBitmapWidth)) -mViewWidth;

       float eventX = event.getX();
       float eventY = event.getY();
       float spacingX = eventX - mStartPoint.x;
       float spacingY = eventY - mStartPoint.y;
       float newPositionLeft = (left  < 0 ? spacingX : spacingX * -1) + left;
       float newPositionRight = (spacingX) + right;
       float newPositionTop = (top  < 0 ? spacingY : spacingY * -1) + top;
       float newPositionBottom = (spacingY) + bottom;
       boolean x = true;
       boolean y = true;

       if(newPositionRight < 0.0f || newPositionLeft > 0.0f){
           if(newPositionRight < 0.0f && newPositionLeft > 0.0f){
               x = false;
           } else{
               eventX = oldEventX;
               mStartPoint.x = oldStartPointX;
           }
       }
       if(newPositionBottom < 0.0f || newPositionTop > 0.0f){
           if(newPositionBottom < 0.0f && newPositionTop > 0.0f){
               y = false;
           } else{
               eventY = oldEventY;
               mStartPoint.y = oldStartPointY;
           }
       }

       if(mDraggable){
           matrix.set(savedMatrix);
           matrix.postTranslate(x? eventX - mStartPoint.x : 0, y? eventY - mStartPoint.y : 0);
           this.setImageMatrix(matrix);
           if(x)oldEventX = eventX;
           if(y)oldEventY = eventY;
           if(x)oldStartPointX = mStartPoint.x;
           if(y)oldStartPointY = mStartPoint.y;
       }

   }

   public void zoom(MotionEvent event){
       matrix.getValues(matrixValues);

       float newDist = spacing(event);
       float bitmapWidth = matrixValues[0] * mBitmapWidth;
       float bimtapHeight = matrixValues[0] * mBitmapHeight;
       boolean in = newDist > oldDist;

       if(!in && matrixValues[0] < 1){
           return;
       }
       if(bitmapWidth > mViewWidth || bimtapHeight > mViewHeight){
           mDraggable = true;
       } else{
           mDraggable = false;
       }

       float midX = (mViewWidth / 2);
       float midY = (mViewHeight / 2);

       matrix.set(savedMatrix);
       scale = newDist / oldDist;
       matrix.postScale(scale, scale, bitmapWidth > mViewWidth ? mMiddlePoint.x : midX, bimtapHeight > mViewHeight ? mMiddlePoint.y : midY); 

       this.setImageMatrix(matrix);


   }





    /** Determine the space between the first two fingers */
    private float spacing(MotionEvent event) {
        float x = event.getX(0) - event.getX(1);
        float y = event.getY(0) - event.getY(1);

        return (float)Math.sqrt(x * x + y * y);
    }

    /** Calculate the mid point of the first two fingers */
    private void midPoint(PointF point, MotionEvent event) {
        float x = event.getX(0) + event.getX(1);
        float y = event.getY(0) + event.getY(1);
        point.set(x / 2, y / 2);
    }


}

This is how you can use it in your activity:

CustomImageVIew mImageView = (CustomImageVIew)findViewById(R.id.customImageVIew1);
mImage.setBitmap(your bitmap);

And layout:

<your.package.name.CustomImageVIew
        android:id="@+id/customImageVIew1"
        android:layout_width="fill_parent"
        android:layout_height="fill_parent"
        android:layout_marginBottom="15dp"
        android:layout_marginLeft="15dp"
        android:layout_marginRight="15dp"
        android:layout_marginTop="15dp"
        android:layout_centerHorizontal="true"
        android:layout_centerVertical="true" 
        android:scaleType="matrix"/> // important

How do you cache an image in Javascript

I use a similar technique to lazyload images, but can't help but notice that Javascript doesn't access the browser cache on first loading.

My example:

I have a rotating banner on my homepage with 4 images the slider wait 2 seconds, than the javascript loads the next image, waits 2 seconds, etc.

These images have unique urls that change whenever I modify them, so they get caching headers that will cache in the browser for a year.

max-age: 31536000, public

Now when I open Chrome Devtools and make sure de 'Disable cache' option is not active and load the page for the first time (after clearing the cache) all images get fetch and have a 200 status. After a full cycle of all images in the banner the network requests stop and the cached images are used.

Now when I do a regular refresh or go to a subpage and click back, the images that are in the cache seems to be ignored. I would expect to see a grey message "from disk cache" in the Network tab of Chrome devtools. In instead I see the requests pass by every two seconds with a Green status circle instead of gray, I see data being transferred, so it I get the impression the cache is not accessed at all from javascript. It simply fetches the image each time the page gets loaded.

So each request to the homepage triggers 4 requests regardless of the caching policy of the image.

Considering the above together and the new http2 standard most webservers and browsers now support, I think it's better to stop using lazyloading since http2 will load all images nearly simultaneously.

If this is a bug in Chrome Devtools it really surprises my nobody noticed this yet. ;)

If this is true, using lazyloading only increases bandwith usage.

Please correct me if I'm wrong. :)

how to run a command at terminal from java program?

You don't actually need to run a command from an xterm session, you can run it directly:

String[] arguments = new String[] {"/path/to/executable", "arg0", "arg1", "etc"};
Process proc = new ProcessBuilder(arguments).start();

If the process responds interactively to the input stream, and you want to inject values, then do what you did before:

OutputStream out = proc.getOutputStream();  
out.write("command\n");  
out.flush();

Don't forget the '\n' at the end though as most apps will use it to identify the end of a single command's input.

In practice, what are the main uses for the new "yield from" syntax in Python 3.3?

This code defines a function fixed_sum_digits returning a generator enumerating all six digits numbers such that the sum of digits is 20.

def iter_fun(sum, deepness, myString, Total):
    if deepness == 0:
        if sum == Total:
            yield myString
    else:  
        for i in range(min(10, Total - sum + 1)):
            yield from iter_fun(sum + i,deepness - 1,myString + str(i),Total)

def fixed_sum_digits(digits, Tot):
    return iter_fun(0,digits,"",Tot) 

Try to write it without yield from. If you find an effective way to do it let me know.

I think that for cases like this one: visiting trees, yield from makes the code simpler and cleaner.

Unable to create/open lock file: /data/mongod.lock errno:13 Permission denied

Every time you when you try to start mongod just type

sudo mongod

or if permanently want to fix this just try to give rwx premission to /data/db folder

 chmod +rwx data/

Sublime 3 - Set Key map for function Goto Definition

For anyone else who wants to set Eclipse style goto definition, you need to create .sublime-mousemap file in Sublime User folder.

Windows - create Default (Windows).sublime-mousemap in %appdata%\Sublime Text 3\Packages\User

Linux - create Default (Linux).sublime-mousemap in ~/.config/sublime-text-3/Packages/User

Mac - create Default (OSX).sublime-mousemap in ~/Library/Application Support/Sublime Text 3/Packages/User

Now open that file and put the following configuration inside

[
    {
        "button": "button1", 
        "count": 1, 
        "modifiers": ["ctrl"],
        "press_command": "drag_select",
        "command": "goto_definition"
    }
]

You can change modifiers key as you like.


Since Ctrl-button1 on Windows and Linux is used for multiple selections, adding a second modifier key like Alt might be a good idea if you want to use both features:

[
    {
        "button": "button1", 
        "count": 1, 
        "modifiers": ["ctrl", "alt"],
        "press_command": "drag_select",
        "command": "goto_definition"
    }
]

Alternatively, you could use the right mouse button (button2) with Ctrl alone, and not interfere with any built-in functions.

How to set <iframe src="..."> without causing `unsafe value` exception?

constructor(
 public sanitizer: DomSanitizer, ) {

 }

I had been struggling for 4 hours. the problem was in img tag. When you use square bracket to 'src' ex: [src]. you can not use this angular expression {{}}. you just give directly from an object example below. if you give angular expression {{}}. you will get interpolation error.

  1. first i used ngFor to iterate the countries

    *ngFor="let country of countries"
    
  2. second you put this in the img tag. this is it.

    <img [src]="sanitizer.bypassSecurityTrustResourceUrl(country.flag)"
    height="20" width="20" alt=""/>
    

Adding JPanel to JFrame

Instead of having your Test2 class contain a JPanel, you should have it subclass JPanel:

public class Test2 extends JPanel {

Test2(){

...

}

More details:

JPanel is a subclass of Component, so any method that takes a Component as an argument can also take a JPanel as an argument.

Older versions didn't let you add directly to a JFrame; you had to use JFrame.getContentPane().add(Component). If you're using an older version, this might also be an issue. Newer versions of Java do let you call JFrame.add(Component) directly.

How to correctly set the ORACLE_HOME variable on Ubuntu 9.x?

Once I also got that same type of error.

I.E:

C:\oracle\product\10.2.0\db_2>SQLPLUS SYS AS SYSDBA
Error 6 initializing SQL*Plus
Message file sp1<lang>.msb not found
SP2-0750: You may need to set ORACLE_HOME to your Oracle software directory

This error is occurring as the home path is not correctly set. To rectify this, if you are using Windows, run the below query:

C:\oracle\product\10.2.0\db_2>SET ORACLE_HOME=C:\oracle\product\10.2.0\db_2
C:\oracle\product\10.2.0\db_2>SQLPLUS SYS AS SYSDBA

SQL*Plus: Release 10.2.0.3.0 - Production on Tue Apr 16 13:17:42 2013

Copyright (c) 1982, 2006, Oracle.  All Rights Reserved.

Or if you are using Linux, then replace set with export for the above command like so:

C:\oracle\product\10.2.0\db_2>EXPORT ORACLE_HOME='C:\oracle\product\10.2.0\db_2'
C:\oracle\product\10.2.0\db_2>SQLPLUS SYS AS SYSDBA

SQL*Plus: Release 10.2.0.3.0 - Production on Tue Apr 16 13:17:42 2013

Copyright (c) 1982, 2006, Oracle.  All Rights Reserved.

Adding div element to body or document in JavaScript

Using Javascript

var elemDiv = document.createElement('div');
elemDiv.style.cssText = 'position:absolute;width:100%;height:100%;opacity:0.3;z-index:100;background:#000;';
document.body.appendChild(elemDiv);

Using jQuery

$('body').append('<div style="position:absolute;width:100%;height:100%;opacity:0.3;z-index:100;background:#000;"></div>');

Check if $_POST exists

Try

if (isset($_POST['fromPerson']) && $_POST['fromPerson'] != "") {
    echo "Cool";
}

Handling errors in Promise.all

Promise.all is all or nothing. It resolves once all promises in the array resolve, or reject as soon as one of them rejects. In other words, it either resolves with an array of all resolved values, or rejects with a single error.

Some libraries have something called Promise.when, which I understand would instead wait for all promises in the array to either resolve or reject, but I'm not familiar with it, and it's not in ES6.

Your code

I agree with others here that your fix should work. It should resolve with an array that may contain a mix of successful values and errors objects. It's unusual to pass error objects in the success-path but assuming your code is expecting them, I see no problem with it.

The only reason I can think of why it would "not resolve" is that it's failing in code you're not showing us and the reason you're not seeing any error message about this is because this promise chain is not terminated with a final catch (as far as what you're showing us anyway).

I've taken the liberty of factoring out the "existing chain" from your example and terminating the chain with a catch. This may not be right for you, but for people reading this, it's important to always either return or terminate chains, or potential errors, even coding errors, will get hidden (which is what I suspect happened here):

Promise.all(state.routes.map(function(route) {
  return route.handler.promiseHandler().catch(function(err) {
    return err;
  });
}))
.then(function(arrayOfValuesOrErrors) {
  // handling of my array containing values and/or errors. 
})
.catch(function(err) {
  console.log(err.message); // some coding error in handling happened
});

jQuery find and replace string

var string ='my string'
var new_string = string.replace('string','new string');
alert(string);
alert(new_string);

Display animated GIF in iOS

#import <QuickLook/QuickLook.h>
#import "ViewController.h"

@implementation ViewController

- (void)viewDidLoad
{
    [super viewDidLoad];

    QLPreviewController *preview = [[QLPreviewController alloc] init];
    preview.dataSource = self;

    [self addChildViewController:preview];
    [self.view addSubview:preview.view];
}

#pragma mark - QLPreviewControllerDataSource

- (NSInteger)numberOfPreviewItemsInPreviewController:(QLPreviewController *)previewController
{
    return 1;
}

- (id)previewController:(QLPreviewController *)previewController previewItemAtIndex:(NSInteger)idx
{
    NSURL *fileURL = [NSURL fileURLWithPath:[[NSBundle mainBundle] pathForResource:@"myanimated.gif" ofType:nil]];
    return fileURL;
}

@end

How can I add a vertical scrollbar to my div automatically?

You have to add max-height property.

_x000D_
_x000D_
.ScrollStyle_x000D_
{_x000D_
    max-height: 150px;_x000D_
    overflow-y: scroll;_x000D_
}
_x000D_
<div class="ScrollStyle">_x000D_
  Scrollbar Test!<br/>_x000D_
  Scrollbar Test!<br/>_x000D_
  Scrollbar Test!<br/>_x000D_
  Scrollbar Test!<br/>_x000D_
  Scrollbar Test!<br/>_x000D_
  Scrollbar Test!<br/>_x000D_
  Scrollbar Test!<br/>_x000D_
  Scrollbar Test!<br/>_x000D_
  Scrollbar Test!<br/>_x000D_
  Scrollbar Test!<br/>_x000D_
</div>
_x000D_
_x000D_
_x000D_

XAMPP: Couldn't start Apache (Windows 10)

Update: 15th May, 2018:

The latest Windows 10 update (re-)activated the World Wide Web Publishing Service (in German: WWW-Publishingdienst). This might depend on the options you select during the configuration of the update you can make afterwards.

Update: 4th August, 2015:

If you have done clean installation of Windows 10, you may not have the Word Wide Web Publishing Service. In that case, simple WAMP/XAMPP installation should work fine.

If it doesn't, try installing Visual C++ Redistributable and then re-install WAMP/XAMPP.


I was facing a similar problem with WAMP. In Windows 10 TP, the Word Wide Web Publishing Service comes pre-installed. This is related to IIS and you can remove it if you don't need it.

This blocks the port 80, making Apache act weirdly. You can do the following and try again.

  • Go to Start, type in services.msc
  • Scroll down in the Services window to find the World Wide Web Publishing Service.
  • Right click on it and select Stop.

This should make port 80 free and restarting WAMP/XAMPP should get you up and running!

There are other ways to do fix this. See Make WAMP Work On Windows 10.

When to favor ng-if vs. ng-show/ng-hide?

ng-if on ng-include and on ng-controller will have a big impact matter on ng-include it will not load the required partial and does not process unless flag is true on ng-controller it will not load the controller unless flag is true but the problem is when a flag gets false in ng-if it will remove from DOM when flag gets true back it will reload the DOM in this case ng-show is better, for one time show ng-if is better

How do I debug "Error: spawn ENOENT" on node.js?

For ENOENT on Windows, https://github.com/nodejs/node-v0.x-archive/issues/2318#issuecomment-249355505 fix it.

e.g. replace spawn('npm', ['-v'], {stdio: 'inherit'}) with:

  • for all node.js version:

    spawn(/^win/.test(process.platform) ? 'npm.cmd' : 'npm', ['-v'], {stdio: 'inherit'})
    
  • for node.js 5.x and later:

    spawn('npm', ['-v'], {stdio: 'inherit', shell: true})
    

How do I increase modal width in Angular UI Bootstrap?

I found it easier to just take over the template from Bootstrap-ui. I have left the commented HTML still in-place to show what I changed.

Overwrite their default template:

<script type="text/ng-template" id="myDlgTemplateWrapper.html">
    <div tabindex="-1" role="dialog" class="modal fade" ng-class="{in: animate}" 
         ng-style="{'z-index': 1050 + index*10, display: 'block'}" ng-click="close($event)"> 
        <!-- <div class="modal-dialog"
            ng-class="{'modal-sm': size == 'sm', 'modal-lg': size == 'lg'}"
            >
            <div class="modal-content"  modal-transclude></div>
        </div>--> 

        <div modal-transclude>
            <!-- Your content goes here -->
        </div>
    </div>
</script>

Modify your dialog template (note the wrapper DIVs containing "modal-dialog" class and "modal-content" class):

<script type="text/ng-template" id="myModalContent.html">
    <div class="modal-dialog {{extraDlgClass}}"
         style="width: {{width}}; max-width: {{maxWidth}}; min-width: {{minWidth}}; ">
        <div class="modal-content">
            <div class="modal-header bg-primary">
                <h3>I am a more flexible modal!</h3>
            </div>
            <div class="modal-body"
                 style="min-height: {{minHeight}}; height: {{height}}; max-height {{maxHeight}}; ">
                <p>Make me any size you want</p>
            </div>
            <div class="modal-footer">
                <button class="btn btn-primary" ng-click="ok()">OK</button>
                <button class="btn btn-warning" ng-click="cancel()">Cancel</button>
            </div>
        </div>
    </div>
</script>

And then call the modal with whatever CSS class or style parameters you wish to change (assuming you have already defined "app" somewhere else):

<script type="text/javascript">
    app.controller("myTest", ["$scope", "$modal", function ($scope, $modal)
    {
        //  Adjust these with your parameters as needed

        $scope.extraDlgClass = undefined;

        $scope.width = "70%";
        $scope.height = "200px";
        $scope.maxWidth = undefined;
        $scope.maxHeight = undefined;
        $scope.minWidth = undefined;
        $scope.minHeight = undefined;

        $scope.open = function ()
        {
            $scope.modalInstance = $modal.open(
            {
                backdrop: 'static',
                keyboard: false,
                modalFade: true,

                templateUrl: "myModalContent.html",
                windowTemplateUrl: "myDlgTemplateWrapper.html",
                scope: $scope,
                //size: size,   - overwritten by the extraDlgClass below (use 'modal-lg' or 'modal-sm' if desired)

                extraDlgClass: $scope.extraDlgClass,

                width: $scope.width,
                height: $scope.height,
                maxWidth: $scope.maxWidth,
                maxHeight: $scope.maxHeight,
                minWidth: $scope.minWidth,
                minHeight: $scope.minHeight
            });

            $scope.modalInstance.result.then(function ()
            {
                console.log('Modal closed at: ' + new Date());
            },
            function ()
            {
                console.log('Modal dismissed at: ' + new Date());
            });
        };

        $scope.ok = function ($event)
        {
            if ($event)
                $event.preventDefault();
            $scope.modalInstance.close("OK");
        };

        $scope.cancel = function ($event)
        {
            if ($event)
                $event.preventDefault();
            $scope.modalInstance.dismiss('cancel');
        };

        $scope.openFlexModal = function ()
        {
            $scope.open();
        }

    }]);
</script>

Add an "open" button and fire away.

    <button ng-controller="myTest" class="btn btn-default" type="button" ng-click="openFlexModal();">Open Flex Modal!</button>

Now you can add whatever extra class you want, or simply change width/height sizes as necessary.

I further enclosed it within a wrapper directive, which is should be trivial from this point forward.

Cheers.

Android studio Gradle icon error, Manifest Merger

Just add xmlns:tools="http://schemas.android.com/tools" to your manifest tag, and then you need to add tools:replace="android:icon" before android:icon="@mipmap/ic_launcher".

sql primary key and index

I have a huge database with no (separate) index.

Any time I query by the primary key the results are, for all intensive purposes, instant.

How do I convert ticks to minutes?

A single tick represents one hundred nanoseconds or one ten-millionth of a second. FROM MSDN.

So 28 000 000 000 * 1/10 000 000 = 2 800 sec. 2 800 sec /60 = 46.6666min

Or you can do it programmaticly with TimeSpan:

    static void Main()
    {
        TimeSpan ts = TimeSpan.FromTicks(28000000000);
        double minutesFromTs = ts.TotalMinutes;
        Console.WriteLine(minutesFromTs);
        Console.Read();
    }

Both give me 46 min and not 480 min...

Eclipse: Java was started but returned error code=13

Like Vito mentions, this error occurs after Java updates as the path:

C:\ProgramData\Oracle\Java\javapath

is added to the Path environment variable, causing Eclipse to run using the wrong java version.

To fix the problem:

1) Right-click on Computer and choose Properties.

2) Click Advanced system settings

3) Click Environment Variables...

4) Find the Path variable in the System variables section.

5) Choose it and click Edit...

6) Find and delete the above mentioned path.

This fixed it for me. I should mention that I already have the path:

c:\Program Files\Java\jdk1.7.0_21\bin

in the Path variable, but the new path was added to the beginning of the Path variable and therefore resolution would use that path first.

Instantiating a generic class in Java

I could do this in a JUnit Test Setup.

I wanted to test a Hibernate facade so I was looking for a generic way to do it. Note that the facade also implements a generic interface. Here T is the database class and U the primary key. Ifacade<T,U> is a facade to access the database object T with the primary key U.

public abstract class GenericJPAController<T, U, C extends IFacade<T,U>>

{
    protected static EntityManagerFactory emf;

    /* The properties definition is straightforward*/
    protected T testObject;
    protected C facadeManager;

    @BeforeClass
    public static void setUpClass() {


        try {
            emf = Persistence.createEntityManagerFactory("my entity manager factory");

        } catch (Throwable ex) {
            System.err.println("Failed to create sessionFactory object." + ex);
            throw new ExceptionInInitializerError(ex);
        }

    }

    @AfterClass
    public static void tearDownClass() {
    }

    @Before
    public void setUp() {
    /* Get the class name*/
        String className = ((ParameterizedType) getClass().getGenericSuperclass()).getActualTypeArguments()[2].getTypeName();

        /* Create the instance */
        try {
            facadeManager = (C) Class.forName(className).newInstance();
        } catch (ClassNotFoundException | InstantiationException | IllegalAccessException ex) {
            Logger.getLogger(GenericJPAController.class.getName()).log(Level.SEVERE, null, ex);
        }
        createTestObject();
    }

    @After
    public void tearDown() {
    }

    /**
     * Test of testFindTEntities_0args method, of class
     * GenericJPAController<T, U, C extends IFacade<T,U>>.
     * @throws java.lang.ClassNotFoundException
     * @throws java.lang.NoSuchMethodException
     * @throws java.lang.InstantiationException
     * @throws java.lang.IllegalAccessException
     */
    @Test
    public void  testFindTEntities_0args() throws ClassNotFoundException, NoSuchMethodException, InstantiationException, IllegalAccessException {

        /* Example of instance usage. Even intellisense (NetBeans) works here!*/
        try {
            List<T> lista = (List<T>) facadeManager.findAllEntities();
            lista.stream().forEach((ct) -> {
                System.out.println("Find all: " + stringReport());
            });
        } catch (Throwable ex) {
            System.err.println("Failed to access object." + ex);
            throw new ExceptionInInitializerError(ex);
        }
    }


    /**
     *
     * @return
     */
    public abstract String stringReport();

    protected abstract T createTestObject();
    protected abstract T editTestObject();
    protected abstract U getTextObjectIndex();
}

PHP header() redirect with POST variables

Use a smarty template for your stuff then just set the POST array as a smarty array and open the template. In the template just echo out the array so if it passes:

if(correct){
    header("Location: passed.php");
} else {
    $smarty->assign("variables", $_POST);
    $smarty->display("register_error.php");
    exit;
}

I have not tried this yet but I am going to try it as a solution and will let you know what I find. But of course this method assumes that you are using smarty.

If not you can just recreate your form there on the error page and echo info into the form or you could send back non important data in a get from and get it

ex.

register.php?name=mr_jones&address==......
echo $_GET[name];

How to delete an SMS from the inbox in Android programmatically?

Just turn off notifications for the default sms app. Process your own notifications for all text messages!

How can I wait for set of asynchronous callback functions?

Use an control flow library like after

after.map(array, function (value, done) {
    // do something async
    setTimeout(function () {
        // do something with the value
        done(null, value * 2)
    }, 10)
}, function (err, mappedArray) {
    // all done, continue here
    console.log(mappedArray)
})

Select from one table where not in another

You can LEFT JOIN the two tables. If there is no corresponding row in the second table, the values will be NULL.

SELECT id FROM partmaster LEFT JOIN product_details ON (...) WHERE product_details.part_num IS NULL

Angular 4 setting selected option in Dropdown

If you want to select a value as default, in your form builder give it a value :

this.myForm = this.FB.group({
    mySelect: [this.options[0].key, [/* Validators here */]]
});

Now in your HTML :

<form [formGroup]="myForm">
    <select [formControlName]="mySelect">
        <option *ngFor="let opt of options" [value]="opt.key">ANY TEXT YOU WANT HERE</option>
    </select>
</form>

What my code does is giving your select a value, that is equal to the first value of your options list. This is how you select an option as default in Angular, selected is useless.

Comparing floating point number to zero

notice, that code is:

std::abs((x - y)/x) <= epsilon

you are requiring that the "relative error" on the var is <= epsilon, not that the absolute difference is

Getting PEAR to work on XAMPP (Apache/MySQL stack on Windows)

On Windows use the Xampp shell (there is a 'Shell' button in your XAMPP control panel)

then

cd php\pear

to go to 'C:\xampp\php\pear'

then type

pear

Resize jqGrid when browser is resized?

<script>
$(document).ready(function(){    
$(window).on('resize', function() {
    jQuery("#grid").setGridWidth($('#fill').width(), false); 
    jQuery("#grid").setGridHeight($('#fill').height(),true); 
}).trigger('resize');      
});
</script>

Integrity constraint violation: 1452 Cannot add or update a child row:

If you are adding new foreign key to an existing table and the columns are not null and not assigned default value, you will get this error,

Either you need to make it nullable or assign default value, or delete all the existing records to solve it.

Purpose of ESI & EDI registers?

In addition to the registers being used for mass operations, they are useful for their property of being preserved through a function call (call-preserved) in 32-bit calling convention. The ESI, EDI, EBX, EBP, ESP are call-preserved whereas EAX, ECX and EDX are not call-preserved. Call-preserved registers are respected by C library function and their values persist through the C library function calls.

Jeff Duntemann in his assembly language book has an example assembly code for printing the command line arguments. The code uses esi and edi to store counters as they will be unchanged by the C library function printf. For other registers like eax, ecx, edx, there is no guarantee of them not being used by the C library functions.

https://www.amazon.com/Assembly-Language-Step-Step-Programming/dp/0470497025

See section 12.8 How C sees Command-Line Arguments.

Note that 64-bit calling conventions are different from 32-bit calling conventions, and I am not sure if these registers are call-preserved or not.

How to compile multiple java source files in command line

Try the following:

javac file1.java file2.java

importing go files in same folder

./main.go (in package main)
./a/a.go (in package a)
./a/b.go (in package a)

in this case:
main.go import "./a"

It can call the function in the a.go and b.go,that with first letter caps on.

gradlew: Permission Denied

I got the same error trying to execute flutter run on a mac. Apparently, in your flutter project, there is a file android/gradlew that is expected to be executable (and it wasn't). So in my case,

chmod a+rx android/gradlew

i used this command and execute the project

Javascript change Div style

function abc() {
    var color = document.getElementById("test").style.color;
    if (color === "red")
         document.getElementById("test").style.color="black";
    else
         document.getElementById("test").style.color="red";
}

How do I center content in a div using CSS?

Update 2020:

There are several options available*:

*Disclaimer: This list may not be complete.

Using Flexbox
Nowadays, we can use flexbox. It is quite a handy alternative to the css-transform option. I would use this solution almost always. If it is just one element maybe not, but for example if I had to support an array of data e.g. rows and columns and I want them to be relatively centered in the very middle.

_x000D_
_x000D_
.flexbox {
  display: flex;
  height: 100px;
  flex-flow: row wrap;
  align-items: center;
  justify-content: center;
  background-color: #eaeaea;
  border: 1px dotted #333;
}

.item {
  /* default => flex: 0 1 auto */
  background-color: #fff;
  border: 1px dotted #333;
  box-sizing: border-box;
}
_x000D_
<div class="flexbox">
  <div class="item">I am centered in the middle.</div>
  <div class="item">I am centered in the middle, too.</div>
</div>
_x000D_
_x000D_
_x000D_


Using CSS 2D-Transform
This is still a good option, was also the accepted solution back in 2015. It is very slim and simple to apply and does not mess with the layouting of other elements.

_x000D_
_x000D_
.boxes {
  position: relative;
}

.box {
  position: relative;
  display: inline-block;
  float: left;
  width: 200px;
  height: 200px;
  font-weight: bold;
  color: #333;
  margin-right: 10px;
  margin-bottom: 10px;
  background-color: #eaeaea;
}

.h-center {
  text-align: center;
}

.v-center span {
  position: absolute;
  left: 0;
  right: 0;
  top: 50%;
  transform: translate(0, -50%);
}
_x000D_
<div class="boxes">
  <div class="box h-center">horizontally centered lorem ipsun dolor sit amet</div>
  <div class="box v-center"><span>vertically centered lorem ipsun dolor sit amet lorem ipsun dolor sit amet</span></div>
  <div class="box h-center v-center"><span>horizontally and vertically centered lorem ipsun dolor sit amet</span></div>
</div>
_x000D_
_x000D_
_x000D_

Note: This does also work with :after and :before pseudo-elements.


Using Grid
This might just be an overkill, but it depends on your DOM. If you want to use grid anyway, then why not. It is very powerful alternative and you are really maximum flexible with the design.

Note: To align the items vertically we use flexbox in combination with grid. But we could also use display: grid on the items.

_x000D_
_x000D_
.grid {
  display: grid;
  width: 400px;
  grid-template-rows: 100px;
  grid-template-columns: 100px 100px 100px;
  grid-gap: 3px;
  align-items: center;
  justify-content: center;
  background-color: #eaeaea;
  border: 1px dotted #333;
}

.item {
  display: flex;
  justify-content: center;
  align-items: center;
  border: 1px dotted #333;
  box-sizing: border-box;
}

.item-large {
  height: 80px;
}
_x000D_
<div class="grid">
  <div class="item">Item 1</div>
  <div class="item item-large">Item 2</div>
  <div class="item">Item 3</div>
</div>
_x000D_
_x000D_
_x000D_


Further reading:

CSS article about grid
CSS article about flexbox
CSS article about centering without flexbox or grid

Filter df when values matches part of a string in pyspark

When filtering a DataFrame with string values, I find that the pyspark.sql.functions lower and upper come in handy, if your data could have column entries like "foo" and "Foo":

import pyspark.sql.functions as sql_fun
result = source_df.filter(sql_fun.lower(source_df.col_name).contains("foo"))

Activate tabpage of TabControl

You can use the method SelectTab.

There are 3 versions:

public void SelectTab(int index);
public void SelectTab(string tabPageName);
public void SelectTab(TabPage tabPage);

At least one JAR was scanned for TLDs yet contained no TLDs

(tomcat 7.0.32) I had problems to see debug messages althought was enabling TldLocationsCache row in tomcat/conf/logging.properties file. All I could see was a warning but not what libs were scanned. Changed every loglevel tried everything no luck. Then I went rogue debug mode (=remove one by one, clean install etc..) and finally found a reason.

My webapp had a customized tomcat/webapps/mywebapp/WEB-INF/classes/logging.properties file. I copied TldLocationsCache row to this file, finally I could see jars filenames.

# To see debug messages in TldLocationsCache, uncomment the following line: org.apache.jasper.compiler.TldLocationsCache.level = FINE

Converting String to Cstring in C++

string name;
char *c_string;

getline(cin, name);

c_string = new char[name.length()];

for (int index = 0; index < name.length(); index++){
    c_string[index] = name[index];
}
c_string[name.length()] = '\0';//add the null terminator at the end of
                              // the char array

I know this is not the predefined method but thought it may be useful to someone nevertheless.

HTTP vs HTTPS performance

The overhead is NOT due to the encryption. On a modern CPU, the encryption required by SSL is trivial.

The overhead is due to the SSL handshakes, which are lengthy and drastically increase the number of round-trips required for a HTTPS session over a HTTP one.

Measure (using a tool such as Firebug) the page load times while the server is on the end of a simulated high-latency link. Tools exist to simulate a high latency link - for Linux there is "netem". Compare HTTP with HTTPS on the same setup.

The latency can be mitigated to some extent by:

  • Ensuring that your server is using HTTP keepalives - this allows the client to reuse SSL sessions, which avoids the need for another handshake
  • Reducing the number of requests to as few as possible - by combining resources where possible (e.g. .js include files, CSS) and encouraging client-side caching
  • Reduce the number of page loads, e.g. by loading data not required into the page (perhaps in a hidden HTML element) and then showing it using client-script.

How to get the root dir of the Symfony2 application?

Since Symfony 3.3 you can use binding, like

services:
_defaults:
    autowire: true      
    autoconfigure: true
    bind:
        $kernelProjectDir: '%kernel.project_dir%'

After that you can use parameter $kernelProjectDir in any controller OR service. Just like

class SomeControllerOrService
{
    public function someAction(...., $kernelProjectDir)
    {
          .....

How to access form methods and controls from a class in C#?

You need to make the members in the for the form class either public or, if the service class is in the same assembly, internal. Windows controls' visibility can be controlled through their Modifiers properties.

Note that it's generally considered a bad practice to explicitly tie a service class to a UI class. Rather you should create good interfaces between the service class and the form class. That said, for learning or just generally messing around, the earth won't spin off its axis if you expose form members for service classes.

rp

How to create a delay in Swift?

If you need to set a delay of less than a second, it is not necessary to set the .seconds parameter. I hope this is useful to someone.

DispatchQueue.main.asyncAfter(deadline: .now() + 0.5, execute: {
        // your code hear
})

Extract number from string with Oracle function

You'd use REGEXP_REPLACE in order to remove all non-digit characters from a string:

select regexp_replace(column_name, '[^0-9]', '')
from mytable;

or

select regexp_replace(column_name, '[^[:digit:]]', '')
from mytable;

Of course you can write a function extract_number. It seems a bit like overkill though, to write a funtion that consists of only one function call itself.

create function extract_number(in_number varchar2) return varchar2 is
begin
  return regexp_replace(in_number, '[^[:digit:]]', '');
end; 

Remove all elements contained in another array

ECMAScript 6 sets can permit faster computing of the elements of one array that aren't in the other:

_x000D_
_x000D_
const myArray = ['a', 'b', 'c', 'd', 'e', 'f', 'g'];
const toRemove = new Set(['b', 'c', 'g']);

const difference = myArray.filter( x => !toRemove.has(x) );

console.log(difference); // ["a", "d", "e", "f"]
_x000D_
_x000D_
_x000D_

Since the lookup complexity for the V8 engine browsers use these days is O(1), the time complexity of the whole algorithm is O(n).

Java creating .jar file

way 1 :

Let we have java file test.java which contains main class testa now first we compile our java file simply as javac test.java we create file manifest.txt in same directory and we write Main-Class: mainclassname . e.g :

  Main-Class: testa

then we create jar file by this command :

  jar cvfm anyname.jar manifest.txt testa.class

then we run jar file by this command : java -jar anyname.jar

way 2 :

Let we have one package named one and every class are inside it. then we create jar file by this command :

  jar cf anyname.jar one

then we open manifest.txt inside directory META-INF in anyname.jar file and write

  Main-Class: one.mainclassname 

in third line., then we run jar file by this command :

  java -jar anyname.jar

to make jar file having more than one class file : jar cf anyname.jar one.class two.class three.class......

Get index of a key/value pair in a C# dictionary based on the value

no , there is nothing similar IndexOf for Dictionary although you can make use of ContainsKey method to get whether a key belongs to dictionary or not

Finishing current activity from a fragment

When working with fragments, instead of using this or refering to the context, always use getActivity(). You should call

getActivity().finish();

to finish your activity from fragment.

Can't bind to 'ngModel' since it isn't a known property of 'input'

I upgraded from RC1 to RC5 and received this error.

I completed my migration (introducing a new app.module.ts file, changing package.json to include new versions and missing modules, and finally changing my main.ts to boot accordingly, based on the Angular2 quick start example).

I did an npm update and then an npm outdated to confirm the versions installed were correct, still no luck.

I ended up completely wiping the node_modules folder and reinstalling with npm install - Voila! Problem solved.

"unadd" a file to svn before commit

Full process (Unix svn package):

Check files are not in SVN:

> svn st -u folder 
? folder

Add all (including ignored files):

> svn add folder
A   folder
A   folder/file1.txt
A   folder/folder2
A   folder/folder2/file2.txt
A   folder/folderToIgnore
A   folder/folderToIgnore/fileToIgnore1.txt
A   fileToIgnore2.txt

Remove "Add" Flag to All * Ignore * files:

> cd folder

> svn revert --recursive folderToIgnore
Reverted 'folderToIgnore'
Reverted 'folderToIgnore/fileToIgnore1.txt'


> svn revert fileToIgnore2.txt
Reverted 'fileToIgnore2.txt'

Edit svn ignore on folder

svn propedit svn:ignore .

Add two singles lines with just the following:

folderToIgnore
fileToIgnore2.txt

Check which files will be upload and commit:

> cd ..

> svn st -u
A   folder
A   folder/file1.txt
A   folder/folder2
A   folder/folder2/file2.txt


> svn ci -m "Commit message here"

How to identify if a webpage is being loaded inside an iframe or directly into the browser window?

It's an ancient piece of code that I've used a few times:

if (parent.location.href == self.location.href) {
    window.location.href = 'https://www.facebook.com/pagename?v=app_1357902468';
}

How to change target build on Android project?

The problem sometimes occurs when there are errors in the project.

For instance, if your project is configured with a target of 3.2 but the 3.2 libraries are not available, you will not be able to change the version to 4.0!

The usual (perhaps brutal) solution I use is to create a new project with the correct target and copy src, res and manifest into the new project.

Update:

This seems to work:

  1. Change the selected through the build properties as normal
  2. Manually edit project.properties AND default.properties to make sure they both reflect the desired target.
  3. Close the project and re-open it

I always run Android Tools | Fix Project Properties after making any changes to the build target.

Differences between arm64 and aarch64

AArch64 is the 64-bit state introduced in the Armv8-A architecture (https://en.wikipedia.org/wiki/ARM_architecture#ARMv8-A). The 32-bit state which is backwards compatible with Armv7-A and previous 32-bit Arm architectures is referred to as AArch32. Therefore the GNU triplet for the 64-bit ISA is aarch64. The Linux kernel community chose to call their port of the kernel to this architecture arm64 rather than aarch64, so that's where some of the arm64 usage comes from.

As far as I know the Apple backend for aarch64 was called arm64 whereas the LLVM community-developed backend was called aarch64 (as it is the canonical name for the 64-bit ISA) and later the two were merged and the backend now is called aarch64.

So AArch64 and ARM64 refer to the same thing.

add string to String array

As many of the answer suggesting better solution is to use ArrayList. ArrayList size is not fixed and it is easily manageable.

It is resizable-array implementation of the List interface. Implements all optional list operations, and permits all elements, including null. In addition to implementing the List interface, this class provides methods to manipulate the size of the array that is used internally to store the list.

Each ArrayList instance has a capacity. The capacity is the size of the array used to store the elements in the list. It is always at least as large as the list size. As elements are added to an ArrayList, its capacity grows automatically.

Note that this implementation is not synchronized.

ArrayList<String> scripts = new ArrayList<String>();
scripts.add("test1");
scripts.add("test2");
scripts.add("test3");

Using a dictionary to select function to execute

Often classes are used to enclose methods and following is the extension for answers above with default method in case the method is not found.

class P:

     def p1(self):
         print('Start')

     def p2(self):
         print('Help')

     def ps(self):
         print('Settings')

     def d(self):
         print('Default function')

     myDict = {
         "start": p1,
         "help": p2,
         "settings": ps
     }

     def call_it(self):
         name = 'start'
         f = lambda self, x : self.myDict.get(x, lambda x : self.d())(self)
         f(self, name)


 p = P()
 p.call_it()

How to know elastic search installed version from kibana?

You can Try this, After starting Service of elasticsearch Type below line in your browser.

         localhost:9200

     It will give Output Something like that,

          {
           "status" : 200,
           "name" : "Hypnotia",
           "cluster_name" : "elasticsearch",
           "version" : {
           "number" : "1.7.1",
           "build_hash" : "b88f43fc40b0bcd7f173a1f9ee2e97816de80b19",
           "build_timestamp" : "2015-07-29T09:54:16Z",
           "build_snapshot" : false,
            "lucene_version" : "4.10.4"
                  },
            "tagline" : "You Know, for Search"
                  }

C# nullable string error

For nullable, use ? with all of the C# primitives, except for string.

The following page gives a list of the C# primitives: http://msdn.microsoft.com/en-us/library/aa711900(v=vs.71).aspx

How to hide a TemplateField column in a GridView

GridView1.Columns[columnIndex].Visible = false;

Parse query string in JavaScript

Me too! http://jsfiddle.net/drzaus/8EE8k/

(Note: without fancy nested or duplicate checking)

deparam = (function(d,x,params,p,i,j) {
return function (qs) {
    // start bucket; can't cheat by setting it in scope declaration or it overwrites
    params = {};
    // remove preceding non-querystring, correct spaces, and split
    qs = qs.substring(qs.indexOf('?')+1).replace(x,' ').split('&');
    // march and parse
    for (i = qs.length; i > 0;) {
        p = qs[--i];
        // allow equals in value
        j = p.indexOf('=');
        // what if no val?
        if(j === -1) params[d(p)] = undefined;
        else params[d(p.substring(0,j))] = d(p.substring(j+1));
    }

    return params;
};//--  fn  deparam
})(decodeURIComponent, /\+/g);

And tests:

var tests = {};
tests["simple params"] = "ID=2&first=1&second=b";
tests["full url"] = "http://blah.com/?third=c&fourth=d&fifth=e";
tests['just ?'] = '?animal=bear&fruit=apple&building=Empire State Building&spaces=these+are+pluses';
tests['with equals'] = 'foo=bar&baz=quux&equals=with=extra=equals&grault=garply';
tests['no value'] = 'foo=bar&baz=&qux=quux';
tests['value omit'] = 'foo=bar&baz&qux=quux';

var $output = document.getElementById('output');
function output(msg) {
    msg = Array.prototype.slice.call(arguments, 0).join("\n");
    if($output) $output.innerHTML += "\n" + msg + "\n";
    else console.log(msg);
}
var results = {}; // save results, so we can confirm we're not incorrectly referencing
$.each(tests, function(msg, test) {
    var q = deparam(test);
    results[msg] = q;
    output(msg, test, JSON.stringify(q), $.param(q));
    output('-------------------');
});

output('=== confirming results non-overwrite ===');
$.each(results, function(msg, result) {
    output(msg, JSON.stringify(result));
    output('-------------------');
});

Results in:

simple params
ID=2&first=1&second=b
{"second":"b","first":"1","ID":"2"}
second=b&first=1&ID=2
-------------------
full url
http://blah.com/?third=c&fourth=d&fifth=e
{"fifth":"e","fourth":"d","third":"c"}
fifth=e&fourth=d&third=c
-------------------
just ?
?animal=bear&fruit=apple&building=Empire State Building&spaces=these+are+pluses
{"spaces":"these are pluses","building":"Empire State Building","fruit":"apple","animal":"bear"}
spaces=these%20are%20pluses&building=Empire%20State%20Building&fruit=apple&animal=bear
-------------------
with equals
foo=bar&baz=quux&equals=with=extra=equals&grault=garply
{"grault":"garply","equals":"with=extra=equals","baz":"quux","foo":"bar"}
grault=garply&equals=with%3Dextra%3Dequals&baz=quux&foo=bar
-------------------
no value
foo=bar&baz=&qux=quux
{"qux":"quux","baz":"","foo":"bar"}
qux=quux&baz=&foo=bar
-------------------
value omit
foo=bar&baz&qux=quux
{"qux":"quux","foo":"bar"}   <-- it's there, i swear!
qux=quux&baz=&foo=bar        <-- ...see, jQuery found it
-------------------

ERROR 1064 (42000) in MySQL

Finally got a solution.

First .sql file converts into the UTF8.

Then use this command

mysql -p -u root --default_character_set utf8 test </var/201535.sql

---root is the username

---test is the database name

or

mysql -p -u root test < /var/201535.sql 

---root is the username

---test is the database name

In the shell, what does " 2>&1 " mean?

This is just like passing the error to the stdout or the terminal.

That is, cmd is not a command:

$cmd 2>filename
cat filename

command not found

The error is sent to the file like this:

2>&1

Standard error is sent to the terminal.

Excel VBA Check if directory exists error

To be certain that a folder exists (and not a file) I use this function:

Public Function FolderExists(strFolderPath As String) As Boolean
    On Error Resume Next
    FolderExists = ((GetAttr(strFolderPath) And vbDirectory) = vbDirectory)
    On Error GoTo 0
End Function

It works both, with \ at the end and without.

Difference between "this" and"super" keywords in Java

this is a reference to the object typed as the current class, and super is a reference to the object typed as its parent class.

In the constructor, this() calls a constructor defined in the current class. super() calls a constructor defined in the parent class. The constructor may be defined in any parent class, but it will refer to the one overridden closest to the current class. Calls to other constructors in this way may only be done as the first line in a constructor.

Calling methods works the same way. Calling this.method() calls a method defined in the current class where super.method() will call the same method as defined in the parent class.

CSS Float: Floating an image to the left of the text

Check out this sample: http://jsfiddle.net/Epgvc/1/

I just floated the title to the left and added a clear:both div to the bottom..

How to Select Every Row Where Column Value is NOT Distinct

Just for fun, here's another way:

;with counts as (
    select CustomerName, EmailAddress,
      count(*) over (partition by EmailAddress) as num
    from Customers
)
select CustomerName, EmailAddress
from counts
where num > 1

"relocation R_X86_64_32S against " linking Error

Assuming you are generating a shared library, most probably what happens is that the variant of liblog4cplus.a you are using wasn't compiled with -fPIC. In linux, you can confirm this by extracting the object files from the static library and checking their relocations:

ar -x liblog4cplus.a  
readelf --relocs fileappender.o | egrep '(GOT|PLT|JU?MP_SLOT)'

If the output is empty, then the static library is not position-independent and cannot be used to generate a shared object.

Since the static library contains object code which was already compiled, providing the -fPIC flag won't help.

You need to get ahold of a version of liblog4cplus.a compiled with -fPIC and use that one instead.

Converting HTML to XML

I did found a way to convert (even bad) html into well formed XML. I started to base this on the DOM loadHTML function. However during time several issues occurred and I optimized and added patches to correct side effects.

  function tryToXml($dom,$content) {
    if(!$content) return false;

    // xml well formed content can be loaded as xml node tree
    $fragment = $dom->createDocumentFragment();
    // wonderfull appendXML to add an XML string directly into the node tree!

    // aappendxml will fail on a xml declaration so manually skip this when occurred
    if( substr( $content,0, 5) == '<?xml' ) {
      $content = substr($content,strpos($content,'>')+1);
      if( strpos($content,'<') ) {
        $content = substr($content,strpos($content,'<'));
      }
    }

    // if appendXML is not working then use below htmlToXml() for nasty html correction
    if(!@$fragment->appendXML( $content )) {
      return $this->htmlToXml($dom,$content);
    }

    return $fragment;
  }



  // convert content into xml
  // dom is only needed to prepare the xml which will be returned
  function htmlToXml($dom, $content, $needEncoding=false, $bodyOnly=true) {

    // no xml when html is empty
    if(!$content) return false;

    // real content and possibly it needs encoding
    if( $needEncoding ) {
      // no need to convert character encoding as loadHTML will respect the content-type (only)
      $content =  '<meta http-equiv="Content-Type" content="text/html;charset='.$this->encoding.'">' . $content;
    }

    // return a dom from the content
    $domInject = new DOMDocument("1.0", "UTF-8");
    $domInject->preserveWhiteSpace = false;
    $domInject->formatOutput = true;

    // html type
    try {
      @$domInject->loadHTML( $content );
    } catch(Exception $e){
      // do nothing and continue as it's normal that warnings will occur on nasty HTML content
    }
        // to check encoding: echo $dom->encoding
        $this->reworkDom( $domInject );

    if( $bodyOnly ) {
      $fragment = $dom->createDocumentFragment();

      // retrieve nodes within /html/body
      foreach( $domInject->documentElement->childNodes as $elementLevel1 ) {
       if( $elementLevel1->nodeName == 'body' and $elementLevel1->nodeType == XML_ELEMENT_NODE ) {
         foreach( $elementLevel1->childNodes as $elementInject ) {
           $fragment->insertBefore( $dom->importNode($elementInject, true) );
         }
        }
      }
    } else {
      $fragment = $dom->importNode($domInject->documentElement, true);
    }

    return $fragment;
  }



    protected function reworkDom( $node, $level = 0 ) {

        // start with the first child node to iterate
        $nodeChild = $node->firstChild;

        while ( $nodeChild )  {
            $nodeNextChild = $nodeChild->nextSibling;

            switch ( $nodeChild->nodeType ) {
                case XML_ELEMENT_NODE:
                    // iterate through children element nodes
                    $this->reworkDom( $nodeChild, $level + 1);
                    break;
                case XML_TEXT_NODE:
                case XML_CDATA_SECTION_NODE:
                    // do nothing with text, cdata
                    break;
                case XML_COMMENT_NODE:
                    // ensure comments to remove - sign also follows the w3c guideline
                    $nodeChild->nodeValue = str_replace("-","_",$nodeChild->nodeValue);
                    break;
                case XML_DOCUMENT_TYPE_NODE:  // 10: needs to be removed
                case XML_PI_NODE: // 7: remove PI
                    $node->removeChild( $nodeChild );
                    $nodeChild = null; // make null to test later
                    break;
                case XML_DOCUMENT_NODE:
                    // should not appear as it's always the root, just to be complete
                    // however generate exception!
                case XML_HTML_DOCUMENT_NODE:
                    // should not appear as it's always the root, just to be complete
                    // however generate exception!
                default:
                    throw new exception("Engine: reworkDom type not declared [".$nodeChild->nodeType. "]");
            }
            $nodeChild = $nodeNextChild;
        } ;
    }

Now this also allows to add more html pieces into one XML which I needed to use myself. In general it can be used like this:

        $c='<p>test<font>two</p>';
    $dom=new DOMDocument('1.0', 'UTF-8');

$n=$dom->appendChild($dom->createElement('info')); // make a root element

if( $valueXml=tryToXml($dom,$c) ) {
  $n->appendChild($valueXml);
}
    echo '<pre/>'. htmlentities($dom->saveXml($n)). '</pre>';

In this example '<p>test<font>two</p>' will nicely be outputed in well formed XML as '<info><p>test<font>two</font></p></info>'. The info root tag is added as it will also allow to convert '<p>one</p><p>two</p>' which is not XML as it has not one root element. However if you html does for sure have one root element then the extra root <info> tag can be skipped.

With this I'm getting real nice XML out of unstructured and even corrupted HTML!

I hope it's a bit clear and might contribute to other people to use it.

What is the difference between rb and r+b modes in file objects

On Windows, 'b' appended to the mode opens the file in binary mode, so there are also modes like 'rb', 'wb', and 'r+b'. Python on Windows makes a distinction between text and binary files; the end-of-line characters in text files are automatically altered slightly when data is read or written. This behind-the-scenes modification to file data is fine for ASCII text files, but it’ll corrupt binary data like that in JPEG or EXE files. Be very careful to use binary mode when reading and writing such files. On Unix, it doesn’t hurt to append a 'b' to the mode, so you can use it platform-independently for all binary files.

Source: Reading and Writing Files

How can I make a button have a rounded border in Swift?

as aside tip make sure your button is not subclass of any custom class in story board , in such a case your code best place should be in custom class it self cause code works only out of the custom class if your button is subclass of the default UIButton class and outlet of it , hope this may help anyone wonders why corner radios doesn't apply on my button from code .

How do I center an SVG in a div?

Answers above look incomplete as they are talking from css point of view only.

positioning of svg within viewport is affected by two svg attributes

  1. viewBox: defines the rectangle area for svg to cover.
  2. preserveAspectRatio: defined where to place the viewBox wrt viewport and how to strech it if viewport changes.

Follow this link from codepen for detailed description

<svg viewBox="70 160 800 190" preserveAspectRatio="xMaxYMax meet">

Number of visitors on a specific page

Go to Behavior > Site Content > All Pages and put your URI into the search box.enter image description here

SVN- How to commit multiple files in a single shot

I've had no issues committing a few files like this:

svn commit fileDir1/ fileDir2/ -m "updated!"

IIS - can't access page by ip address instead of localhost

So I had this same problem with WSUS and it turned out that IIS 8.5 was not binding to my ipv4 ip address, but was binding to ipv6 address. when I accessed it via localhost:8580 it would translate it to the ipv6 localhost address, and thus would work. Accessing it via ip was a no go. I had to manually bind the address using netsh, then it worked right away. bloody annoying.

Steps:

  1. Open command prompt as administrator
  2. Type the following:

netsh http add iplisten ipaddress (IPADDRESSOFYOURSERVER)

that's it. You should get:

IP address successfully added

I found the commands here https://serverfault.com/questions/123796/get-iis-7-5-to-listen-on-ipv6

Notify ObservableCollection when Item changes

All the solutions here are correct,but they are missing an important scenario in which the method Clear() is used, which doesn't provide OldItems in the NotifyCollectionChangedEventArgs object.

this is the perfect ObservableCollection .

public delegate void ListedItemPropertyChangedEventHandler(IList SourceList, object Item, PropertyChangedEventArgs e);
public class ObservableCollectionEX<T> : ObservableCollection<T>
{
    #region Constructors
    public ObservableCollectionEX() : base()
    {
        CollectionChanged += ObservableCollection_CollectionChanged;
    }
    public ObservableCollectionEX(IEnumerable<T> c) : base(c)
    {
        CollectionChanged += ObservableCollection_CollectionChanged;
    }
    public ObservableCollectionEX(List<T> l) : base(l)
    {
        CollectionChanged += ObservableCollection_CollectionChanged;
    }

    #endregion



    public new void Clear()
    {
        foreach (var item in this)            
            if (item is INotifyPropertyChanged i)                
                i.PropertyChanged -= Element_PropertyChanged;            
        base.Clear();
    }
    private void ObservableCollection_CollectionChanged(object sender, NotifyCollectionChangedEventArgs e)
    {
        if (e.OldItems != null)
            foreach (var item in e.OldItems)                
                if (item != null && item is INotifyPropertyChanged i)                    
                    i.PropertyChanged -= Element_PropertyChanged;


        if (e.NewItems != null)
            foreach (var item in e.NewItems)                
                if (item != null && item is INotifyPropertyChanged i)
                {
                    i.PropertyChanged -= Element_PropertyChanged;
                    i.PropertyChanged += Element_PropertyChanged;
                }
            }
    }
    private void Element_PropertyChanged(object sender, PropertyChangedEventArgs e) => ItemPropertyChanged?.Invoke(this, sender, e);


    public ListedItemPropertyChangedEventHandler ItemPropertyChanged;

}

Click in OK button inside an Alert (Selenium IDE)

This is Pythoncode
Problem with alert boxes (especially sweet-alerts is that they have a delay and Selenium is pretty much too fast)

An Option that worked for me is:

while True:
    try:
        driver.find_element_by_xpath('//div[@class="sweet-alert showSweetAlert visible"]')
        break
    except:
        wait = WebDriverWait(driver, 1000)

confirm_button = driver.find_element_by_xpath('//button[@class="confirm"]')
confirm_button.click()

Check if table exists without using "select from"

SELECT count(*)
FROM information_schema.TABLES
WHERE (TABLE_SCHEMA = 'your_db_name') AND (TABLE_NAME = 'name_of_table')

if you get a non-zero count, the table exists.

Removing a list of characters in string

''.join(c for c in myString if not c in badTokens)

Turn on IncludeExceptionDetailInFaults (either from ServiceBehaviorAttribute or from the <serviceDebug> configuration behavior) on the server

I was also getting the same error, the WCF was working properly for me when i was using it in the Dev Environment with my credentials, but when someone else was using it in TEST, it was throwing the same error. I did a lot of research, and then instead of doing config updates, handled an exception in the WCF method with the help of fault exception. Also the identity for the WCF needs to be set with the same credentials which are having access in the database, someone might have changed your authority. Please find below the code for the same:

 [ServiceContract]
public interface IService1
{
    [OperationContract]
    [FaultContract(typeof(ServiceData))]
    ForDataset GetCCDBdata();

    [OperationContract]
    [FaultContract(typeof(ServiceData))]
    string GetCCDBdataasXMLstring();


    //[OperationContract]
    //string GetData(int value);

    //[OperationContract]
    //CompositeType GetDataUsingDataContract(CompositeType composite);

    // TODO: Add your service operations here
}

  [DataContract]
public class ServiceData
{
    [DataMember]
    public bool Result { get; set; }
    [DataMember]
    public string ErrorMessage { get; set; }
    [DataMember]
    public string ErrorDetails { get; set; }
}

in your service1.svc.cs you can use this in the catch block:

 catch (Exception ex)
        {
            myServiceData.Result = false;
            myServiceData.ErrorMessage = "unforeseen error occured. Please try later.";
            myServiceData.ErrorDetails = ex.ToString();
            throw new FaultException<ServiceData>(myServiceData, ex.ToString());
        }

And use this in the Client application like below code:

  ConsoleApplicationWCFClient.CCDB_HIG_service.ForDataset ds = obj.GetCCDBdata();

            string str = obj.GetCCDBdataasXMLstring();

        }

        catch (FaultException<ConsoleApplicationWCFClient.CCDB_HIG_service.ServiceData> Fex)
      {
          Console.WriteLine("ErrorMessage::" + Fex.Detail.ErrorMessage + Environment.NewLine);
          Console.WriteLine("ErrorDetails::" + Environment.NewLine + Fex.Detail.ErrorDetails);
          Console.ReadLine();
      }

Just try this, it will help for sure to get the exact issue.

Difference between static class and singleton pattern?

Another advantage of a singleton is that it can easily be serialized, which may be necessary if you need to save its state to disc, or send it somewhere remotely.

How do I get the current mouse screen coordinates in WPF?

Do you want coordinates relative to the screen or the application?

If it's within the application just use:

Mouse.GetPosition(Application.Current.MainWindow);

If not, I believe you can add a reference to System.Windows.Forms and use:

System.Windows.Forms.Control.MousePosition;

Hello World in Python

In python 3.x. you use

print("Hello, World")

In Python 2.x. you use

print "Hello, World!"

PHP Create and Save a txt file to root directory

fopen() will open a resource in the same directory as the file executing the command. In other words, if you're just running the file ~/test.php, your script will create ~/myText.txt.

This can get a little confusing if you're using any URL rewriting (such as in an MVC framework) as it will likely create the new file in whatever the directory contains the root index.php file.

Also, you must have correct permissions set and may want to test before writing to the file. The following would help you debug:

$fp = fopen("myText.txt","wb");
if( $fp == false ){
    //do debugging or logging here
}else{
    fwrite($fp,$content);
    fclose($fp);
}

Loop through a date range with JavaScript

Based on Tom Gullen´s answer.

var start = new Date("02/05/2013");
var end = new Date("02/10/2013");


var loop = new Date(start);
while(loop <= end){
   alert(loop);           

   var newDate = loop.setDate(loop.getDate() + 1);
   loop = new Date(newDate);
}

How do I escape double quotes in attributes in an XML String in T-SQL?

In Jelly.core to test a literal string one would use:

&lt;core:when test="${ name == 'ABC' }"&gt; 

But if I have to check for string "Toy's R Us":

&lt;core:when test="${ name == &amp;quot;Toy&apos;s R Us&amp;quot; }"&gt;

It would be like this, if the double quotes were allowed inside:

&lt;core:when test="${ name == "Toy's R Us" }"&gt; 

How do I read a date in Excel format in Python?

A combination of peoples post gave me the date and the time for excel conversion. I did return it as a string

def xldate_to_datetime(xldate):
  tempDate = datetime.datetime(1900, 1, 1)
  deltaDays = datetime.timedelta(days=int(xldate))
  secs = (int((xldate%1)*86400)-60)
  detlaSeconds = datetime.timedelta(seconds=secs)
  TheTime = (tempDate + deltaDays + detlaSeconds )
  return TheTime.strftime("%Y-%m-%d %H:%M:%S")

set height of imageview as matchparent programmatically

imageView.setLayoutParams
    (new ViewGroup.MarginLayoutParams
        (width, ViewGroup.LayoutParams.MATCH_PARENT));

The Type of layout params depends on the parent view group. If you put the wrong one it will cause exception.

What is the correct way to read a serial port using .NET framework?

Could you try something like this for example I think what you are wanting to utilize is the port.ReadExisting() Method

 using System;
 using System.IO.Ports;

 class SerialPortProgram 
 { 
  // Create the serial port with basic settings 
    private SerialPort port = new   SerialPort("COM1",
      9600, Parity.None, 8, StopBits.One); 
    [STAThread] 
    static void Main(string[] args) 
    { 
      // Instatiate this 
      SerialPortProgram(); 
    } 

    private static void SerialPortProgram() 
    { 
        Console.WriteLine("Incoming Data:");
         // Attach a method to be called when there
         // is data waiting in the port's buffer 
        port.DataReceived += new SerialDataReceivedEventHandler(port_DataReceived); 
        // Begin communications 
        port.Open(); 
        // Enter an application loop to keep this thread alive 
        Console.ReadLine();
     } 

    private void port_DataReceived(object sender, SerialDataReceivedEventArgs e) 
    { 
       // Show all the incoming data in the port's buffer
       Console.WriteLine(port.ReadExisting()); 
    } 
}

Or is you want to do it based on what you were trying to do , you can try this

public class MySerialReader : IDisposable
{
  private SerialPort serialPort;
  private Queue<byte> recievedData = new Queue<byte>();

  public MySerialReader()
  {
    serialPort = new SerialPort();
    serialPort.Open();
    serialPort.DataReceived += serialPort_DataReceived;
  }

  void serialPort_DataReceived(object s, SerialDataReceivedEventArgs e)
  {
    byte[] data = new byte[serialPort.BytesToRead];
    serialPort.Read(data, 0, data.Length);
    data.ToList().ForEach(b => recievedData.Enqueue(b));
    processData();
  }

  void processData()
  {
    // Determine if we have a "packet" in the queue
    if (recievedData.Count > 50)
    {
        var packet = Enumerable.Range(0, 50).Select(i => recievedData.Dequeue());
    }
  }

  public void Dispose()
  {
        if (serialPort != null)
        {
            serialPort.Dispose();
        }
  }

When is it appropriate to use C# partial classes?

If you have a sufficiently large class that doesn't lend itself to effective refactoring, separating it into multiple files helps keep things organized.

For instance, if you have a database for a site containing a discussion forum and a products system, and you don't want to create two different providers classes (NOT the same thing as a proxy class, just to be clear), you can create a single partial class in different files, like

MyProvider.cs - core logic

MyProvider.Forum.cs - methods pertaining specifically to the forum

MyProvider.Product.cs - methods for products

It's just another way to keep things organized.

Also, as others have said, it's about the only way to add methods to a generated class without running the risk of having your additions destroyed the next time the class is regenerated. This comes in handy with template-generated (T4) code, ORMs, etc.

Access denied for root user in MySQL command-line

this might help on Ubuntu:

go to /etc/mysql/my.cnf and comment this line:

bind-address           = 127.0.0.1

Hope this helps someone, I've been searching for this a while too

Cheers

How do I get the project basepath in CodeIgniter

Following are the build-in constants you can use as per your requirements for getting the paths in Codeigniter:

EXT: The PHP file extension

FCPATH: Path to the front controller (this file) (root of CI)

SELF: The name of THIS file (index.php)

BASEPATH: Path to the system folder

APPPATH: The path to the “application” folder

Thanks.

Translating touch events from Javascript to jQuery

jQuery 'fixes up' events to account for browser differences. When it does so, you can always access the 'native' event with event.originalEvent (see the Special Properties subheading on this page).

Does hosts file exist on the iPhone? How to change it?

Another option here is to have your iPhone connect via a proxy. Here's an example of how to do it with Fiddler (it's very easy):

http://conceptdev.blogspot.com/2009/01/monitoring-iphone-web-traffic-with.html

In that case any dns lookups your iPhone does will use the hosts file of the machine Fiddler is running on. Note, though, that you must use a name that will be resolved via DNS. example.local, for instance, will not work. example.xyz or example.dev will.

Java: random long number in 0 <= x < n range

The standard method to generate a number (without a utility method) in a range is to just use the double with the range:

long range = 1234567L;
Random r = new Random()
long number = (long)(r.nextDouble()*range);

will give you a long between 0 (inclusive) and range (exclusive). Similarly if you want a number between x and y:

long x = 1234567L;
long y = 23456789L;
Random r = new Random()
long number = x+((long)(r.nextDouble()*(y-x)));

will give you a long from 1234567 (inclusive) through 123456789 (exclusive)

Note: check parentheses, because casting to long has higher priority than multiplication.

How to make an embedded video not autoplay

Try replacing your movie param line with

<param name="movie" value="untitled_skin.swf&autoStart=false">

Netbeans 8.0.2 The module has not been deployed

Try to change Tomcat version, in my case tomcat "8.0.41" and "8.5.8" didn't work. But "8.5.37" worked fine.

Two versions of python on linux. how to make 2.7 the default

I guess you have installed the 2.7 version manually, while 2.6 comes from a package?

The simple answer is: uninstall python package.

The more complex one is: do not install manually in /usr/local. Build a package with 2.7 version and then upgrade.

Package handling depends on what distribution you use.

How do I assert equality on two classes without an equals method?

I tried all the answers and nothing really worked for me.

So I've created my own method that compares simple java objects without going deep into nested structures...

Method returns null if all fields match or string containing mismatch details.

Only properties that have a getter method are compared.

How to use

        assertNull(TestUtils.diff(obj1,obj2,ignore_field1, ignore_field2));

Sample output if there is a mismatch

Output shows property names and respective values of compared objects

alert_id(1:2), city(Moscow:London)

Code (Java 8 and above):

 public static String diff(Object x1, Object x2, String ... ignored) throws Exception{
        final StringBuilder response = new StringBuilder();
        for (Method m:Arrays.stream(x1.getClass().getMethods()).filter(m->m.getName().startsWith("get")
        && m.getParameterCount()==0).collect(toList())){

            final String field = m.getName().substring(3).toLowerCase();
            if (Arrays.stream(ignored).map(x->x.toLowerCase()).noneMatch(ignoredField->ignoredField.equals(field))){
                Object v1 = m.invoke(x1);
                Object v2 = m.invoke(x2);
                if ( (v1!=null && !v1.equals(v2)) || (v2!=null && !v2.equals(v1))){
                    response.append(field).append("(").append(v1).append(":").append(v2).append(")").append(", ");
                }
            }
        }
        return response.length()==0?null:response.substring(0,response.length()-2);
    }

Put Excel-VBA code in module or sheet?

I would suggest separating your code based on the functionality and purpose specific to each sheet or module. In this manner, you would only put code relative to a sheet's UI inside the sheet's module and only put code related to modules in respective modules. Also, use separate modules to encapsulate code that is shared or reused among several different sheets.

For example, let's say you multiple sheets that are responsible for displaying data from a database in a special way. What kinds of functionality do we have in this situation? We have functionality related to each specific sheet, tasks related to getting data from the database, and tasks related to populating a sheet with data. In this case, I might start with a module for the data access, a module for populating a sheet with data, and within each sheet I'd have code for accessing code in those modules.

It might be laid out like this.

Module: DataAccess:

Function GetData(strTableName As String, strCondition1 As String) As Recordset
    'Code Related to getting data from the database'
End Function

Module: PopulateSheet:

Sub PopulateASheet(wsSheet As Worksheet, rs As Recordset)
    'Code to populate a worksheet '
End Function

Sheet: Sheet1 Code:

Sub GetDataAndPopulate()
    'Sample Code'
     Dim rs As New Recordset
     Dim ws As Worksheet
     Dim strParam As String
     Set ws = ActiveSheet
     strParam = ws.Range("A1").Value

     Set rs = GetData("Orders",strParam)

     PopulateASheet ws, rs
End Sub

Sub Button1_Click()
    Call GetDataAndPopulate
End Sub